Evidence Flashcards

ALA quizzes, Kaplan Quizzes, Kaplan Qbank

1
Q

A plaintiff brought suit against a car owner for personal injuries sustained in an automobile accident. The car was driven by a neighbor, who was angry that the plaintiff would not allow him to date the plaintiff’s daughter. Liability is based on a statute that assesses liability on the automobile owner for the negligent actions of those driving with the owner’s consent. The plaintiff offers the testimony of his chiropractor’s receptionist that the plaintiff told her he needed an appointment for the neck and back pain he has had for several days. The car owner’s lawyer objects.

Which of the following is most accurate?

A The court should overrule the objection, because the evidence is admissible as a present sense impression.
B The court should overrule the objection, because the evidence is admissible as a statement for purposes of medical treatment or diagnosis.
C The court should sustain the objection, because a statement of personal history is not appropriate if the declarant is available.
D The court should sustain the objection, because the receptionist is not a medical professional.

A

The correct answer is: The court should overrule the objection, because the evidence is admissible as a statement for purposes of medical treatment or diagnosis.

Discussion of correct answer: The statement appears to be hearsay–an out-of-court statement offered for the truth of the matter asserted. However, it should be admissible under the exception for statements made for the purpose of medical diagnosis or treatment. Note that this exception does not apply only to physicians or other licensed practitioners; it also applies to their agents.

How well did you know this?
1
Not at all
2
3
4
5
Perfectly
2
Q

The defendant is on trial for battery. His defense is self-defense. In his case-in-chief, he offers the testimony of a witness who testifies as follows: “I have known the victim for 25 years and in my opinion the victim is a violent man.” The prosecutor objects to the testimony.

Is the testimony admissible?

A No, because it is improper character evidence.
B No, because it is improper lay opinion evidence.
C Yes, to show the victim acted in conformity with his character for violence.
D Yes, because it is circumstantial evidence of the peaceful character of the defendant.

A

The correct answer is: Yes, to show the victim acted in conformity with his character for violence.

Discussion of correct answer: Although it is the general rule that character evidence is inadmissible, there are instances when character evidence is admissible to prove a relevant character trait of the victim of a battery or some other violent crime with which a defendant is charged, often to support a claim of self-defense. Here, the defendant has offered evidence of a relevant character trait of the victim of the alleged battery. It is admissible in the form of an opinion or reputation evidence. It is a proper offer of character evidence, which the prosecutor is free to rebut, if she so chooses.

How well did you know this?
1
Not at all
2
3
4
5
Perfectly
3
Q

A musical instrument company is sued when one of its delivery trucks is involved in an accident with a passenger car. At trial, the plaintiff asserts that the company failed to properly maintain the brakes on the delivery truck. The company calls its head mechanic to testify that all of the company’s delivery trucks undergo a safety inspection, including an inspection of the brakes, every morning before the trucks leave the yard. He supervises the inspectors, but does not perform the inspections himself, and was not present in the yard on the morning of the accident. Is the head mechanic’s testimony admissible?

a. Yes, because it is evidence of the routine practice of a business.
b. Yes, because it is evidence of the company’s propensity for safe conduct.
c. No, because the head mechanic does not have personal knowledge that the vehicle was inspected that morning.
d. No, because only personal habits are admissible to show that a defendant acted according to habit on a particular occasion.

A

a. Yes, because it is evidence of the routine practice of a business.

How well did you know this?
1
Not at all
2
3
4
5
Perfectly
4
Q

A young husband and wife were going through a divorce. While the divorce was pending, the wife moved out, and the husband remained in the marital home. During the divorce negotiations, the wife sent the husband an email that stated: “You can have all of our furniture, but remember that the bedroom and kitchen furniture belongs to my parents, and they were just letting us use it temporarily.” In the stipulated divorce decree, the wife was awarded the marital home. After the divorce was finalized, the wife returned to the home and discovered that the husband had taken all of the furniture that belonged to her parents. The husband refused to return the furniture, claiming that the furniture was a gift from the wife’s parents and was awarded to him in the divorce. The wife’s parents sued the husband for conversion of the furniture. At trial, the parents’ attorney calls the wife as a witness to testify that the furniture was not a gift to the couple. On cross-examination, the husband’s attorney asks the wife: “Why did you suddenly claim that the furniture belonged to your parents after the divorce was final?”

On redirect, may the parents’ attorney question the wife about the email that the wife sent to the husband during the divorce negotiations?

A No, because the email is inadmissible hearsay.
B No, because the wife is not a party to the litigation.
C Yes, because the email is a recorded recollection.
D Yes, because the wife’s prior consistent statement is not hearsay.

A

The correct answer is: Yes, because the wife’s prior consistent statement is not hearsay.

Discussion of correct answer: A statement is not hearsay when the declarant testifies about the prior statement, and the statement is a prior consistent statement that is offered to rebut a claim that the declarant recently fabricated his/her testimony. The wife testifies at trial that the furniture was not a gift and belonged to her parents. On cross-examination, the husband’s attorney implies that the wife only claimed that the furniture belonged to her parents after the divorce was final. The wife’s email to the husband during the divorce negotiations is a prior statement that is consistent with the wife’s testimony that the furniture was her parents’. Thus, the wife’s prior consistent statement in the email is not hearsay and is admissible to rebut the charge that the wife fabricated the story after the divorce.

How well did you know this?
1
Not at all
2
3
4
5
Perfectly
5
Q

The plaintiff owned a very successful seafood restaurant. At the end of the previous season, the plaintiff became concerned about competition from a new restaurant approximately two miles away, and so he decided to hire a quality-control administrator who would help increase customer satisfaction and profitability. As one of his first new directives, the quality-control administrator created a spreadsheet and instructed the staff to keep tallies of any service-related issues. He had three headings on the spreadsheet: (1) food sent back based on quality; (2) complaints about service in general; and (3) comments about price.

About three months later, the plaintiff learned that his seafood supplier was being paid by the competitor to deliver subpar seafood to the plaintiff and keep the best product for the competitor. The plaintiff then brought suit against the competitor for intentional interference with a business relationship. In order to prove damages, the plaintiff called the quality-control administrator to testify as to the column on the spreadsheet that showed the number of meals that were send back due to food quality.

Is the quality-control administrator allowed to testify as to the spreadsheet?

A No, because the purpose of keeping the record was not sufficiently business-related.
B No, because the spreadsheet constitutes hearsay not within any exception.
C Yes, because the spreadsheet constitutes a past recollection recorded.
D Yes, because the spreadsheet is a business record.

A

The correct answer is: Yes, because the spreadsheet is a business record.

Discussion of correct answer: Hearsay is an out-of-court statement offered into evidence to prove the truth of the matter asserted. Here, the tallies are clearly written testimony being presented for the purpose of proving that the supplier was supplying the plaintiff’s restaurant with inferior seafood. Therefore, the spreadsheet is inadmissible hearsay unless it falls within a recognized hearsay exception. Because the spreadsheet was created for the purpose of improving the plaintiff’s business and not to further his litigation, it was made in the ordinary course of the plaintiff’s business. As such, it comes within the business records exception to the hearsay rule.

How well did you know this?
1
Not at all
2
3
4
5
Perfectly
6
Q

A homeowner purchased a lounge chair from a furniture store. Unfortunately, the chair collapsed while the homeowner was sitting on it, and he suffered a broken arm. The homeowner brought an action against the furniture store for negligence and breach of warranty.

At trial, the homeowner testified that one of the furniture store’s sales representatives told him, “We’re the only retailer of these lounge chairs that sells each chair with a money back guarantee on all parts and workmanship. The furniture store is confident in making this guarantee because specially-trained employees assemble and give each chair a 10-point inspection before delivery to the customer.”

The furniture store’s attorney called the homeowner’s aunt to the stand. On direct examination, the homeowner’s aunt testified that she happened to be at the store when the homeowner purchased the chair, that she did not recall whether or not the salesman made the warranty statement, and that she was not biased against the homeowner. The homeowner’s attorney then questioned the aunt about a bitter family dispute occurring several years ago between the aunt and her sister, the homeowner’s mother. The aunt has not spoken to her sister or her sister’s children, including the homeowner, since the dispute arose. The furniture store’s attorney objects to this line of questions by the homeowner’s attorney. Will the furniture store’s attorney succeed with his objection?

a. Yes, because this testimony addresses a collateral issue.
b. Yes, because this testimony does not have a tendency to prove or disprove any material fact.
c. No, because this testimony tends to establish the aunt’s bias.
d. No, because the testimony would be a statement of family history which fits within an exception to the hearsay rule.

A

c. No, because this testimony tends to establish the aunt’s bias.

How well did you know this?
1
Not at all
2
3
4
5
Perfectly
7
Q

A heartsick lover knew he was being foolish, but he just could not stop himself. The woman he was in love with was so beautiful, and she was so lonely with her husband in prison. She had told the lover that her husband was extremely jealous. She said that her husband was in prison because he had shot her last lover when he learned of their affair. About four months after he started seeing the woman, the lover received a telephone call from her. She seemed distraught, and cried, “My husband escaped from prison this evening, and he knows about us!” As expected, the husband called on the lover that evening. The lover shot the husband and was prosecuted for his murder. The lover claims self-defense. At trial, he attempts to testify to the woman’s statement that her husband was in prison because he had shot her last lover when he learned of the affair.

If the statement is offered to prove that the husband was in prison because he had shot the woman’s lover when he learned of the affair, should the statement be admitted?

A Yes, under the state-of-mind exception.
B Yes, as nonhearsay.
C No, as hearsay not within any exception.
D No, because it is self-serving.

A

The correct answer is: No, as hearsay not within any exception.

Discussion of correct answer: Under FRE 801(c), hearsay is: (1) any statement other than a statement made by the declarant while testifying at the trial or hearing in which the statement is proffered; (2) which is offered into evidence in order to prove the truth of the matter asserted. Here, this is hearsay because the question tells us that the lover offers the statement for the express purpose of proving the truth of the matter asserted, i.e., that the husband had killed his wife’s prior lover. The statement does not fall within any exception, thus it is not admissible.

How well did you know this?
1
Not at all
2
3
4
5
Perfectly
8
Q

The defendant is on trial for battery. His defense is self-defense. In his case-in-chief, he offers the testimony of a witness who testifies as follows: “I have known the victim for 25 years and in my opinion the victim is a violent man.” The prosecutor objects to the testimony. Is the testimony admissible?

a. No, because it is improper character evidence.
b. No, because it is improper lay opinion evidence.
c. Yes, to show the victim acted in conformity with his character for violence.
d. Yes, because it is circumstantial evidence of the peaceful character of the defendant.

A

c. Yes, to show the victim acted in conformity with his character for violence.

How well did you know this?
1
Not at all
2
3
4
5
Perfectly
9
Q

During a farmer’s trial for theft by deception, the State calls a farmhand to testify. The farmhand is the farmer’s co-defendant, who pleaded guilty and received probation for his testimony against the farmer. If the defense attorney has a good-faith basis for the question, which of the following questions may he properly ask the farmhand?

a. Have you ever been convicted of a felony?
b. Have you ever been arrested for a felony?
c. Have you ever been convicted of a misdemeanor?
d. Have you ever been convicted of perjury or disorderly conduct (a misdemeanor)?

A

a. Have you ever been convicted of a felony?

How well did you know this?
1
Not at all
2
3
4
5
Perfectly
10
Q

A criminal defendant was on trial for fraud. The defense called the defendant’s friend as a character witness. He testified that he had known the defendant for 11 years and that he had a good reputation for honesty. After testifying on direct, the prosecution did not cross-examine the witness, and he was excused from the courtroom. On rebuttal, the prosecution sought to admit into evidence an investigative report by the FBI which outlined facts that tended to show that the friend was involved in two unlawful money-laundering operations. The defendant’s attorney objected. Is the report admissible?

a. No, because the witness was not questioned about the matter on cross-examination and was excused after testifying.
b. No, because the report constitutes an improper method of impeachment by attempting to prove specific instances of conduct by extrinsic evidence.
c. Yes, because a witness may be attacked by introducing evidence of prior bad acts that bear directly on dishonesty or untruthfulness.
d. Yes, because the defendant opened the door by calling the friend as a character witness.

A

b. No, because the report constitutes an improper method of impeachment by attempting to prove specific instances of conduct by extrinsic evidence.

How well did you know this?
1
Not at all
2
3
4
5
Perfectly
11
Q

A man was driving to work when his truck collided with a car driven by a woman, who was backing out of her driveway very quickly without looking or yielding to oncoming traffic. The man subsequently filed suit to recover damages from the woman, asserting that he had severely injured his left arm in the accident and that this injury had rendered him unable to work, as he was left-handed. The woman, in turn, claimed that the man’s injury was completely unrelated to the car accident and was the result of repetitive-motion syndrome. The man’s employer required its employees to have an annual examination. The previous year, three months before the accident, the man had visited the doctor for his yearly exam. During the trial, the man testified that his injury had caused him severe pain and left him physically unable to work. He then called the doctor to the stand to testify that the man had told him during the annual exam that his previous arm problems had gone away.

Should the doctor’s testimony be admitted over the woman’s objection?

A No, because it is hearsay not within any exception.
B No, because the man did not make the statement for the purpose of medical treatment.
C Yes, because the man made the statement for the purpose of medical treatment.
D Yes, as a statement by a party-opponent.

A

The correct answer is: Yes, because the man made the statement for the purpose of medical treatment.

Discussion of correct answer: Federal Rule of Evidence 803(4) allows the admission into evidence of a statement made to a healthcare professional for the purpose of a present medical diagnosis or treatment, describing past symptoms or history, as an exception to the rule excluding hearsay evidence. Here, the man visited the doctor for his general physical examination, to ensure that he was healthy and able to work. As such, his statement to the doctor would probably be regarded as one made for the purpose of treatment. Therefore, the statement should be admitted pursuant to this exception to the hearsay rule.

How well did you know this?
1
Not at all
2
3
4
5
Perfectly
12
Q

A realtor is arrested and charged with the arson of a house. On cross-examination of the realtor, the prosecution asked the realtor whether he had been convicted of fraudulent business practices four months earlier. The defense objects to the question. Assuming that the realtor was, in fact, convicted of fraudulent business practices, will the objection be sustained?

a. Yes, because the probative value of the evidence is outweighed by the danger of unfair prejudice.
b. Yes, because the conviction of fraudulent business practices is unrelated to a charge of arson.
c. No, because the conviction can be used to show that the realtor has the propensity to lie.
d. No, because the conviction may serve as character evidence.

A

c. No, because the conviction can be used to show that the realtor has the propensity to lie.

How well did you know this?
1
Not at all
2
3
4
5
Perfectly
13
Q

A girlfriend and boyfriend, out to dinner at a restaurant, got into a heated argument. The boyfriend stood up angrily, and the woman stood up and started to leave. He followed her out, and they continued yelling at each other. They began walking on the street, where she tried to get away from him. She walked faster and faster, but he continued to follow her, yelling and gesticulating angrily. A stranger on the street saw her approaching in a panic. The girlfriend grabbed the stranger in a frenzy and begged for help, saying her boyfriend had just hit her and was chasing her.

The boyfriend was later arrested for assault. At trial, the girlfriend called as witnesses both the stranger and the girlfriend’s own best friend. The stranger planned to testify that the girlfriend had told him that her boyfriend had hit her that night. The best friend, with whom the girlfriend had been staying since the incident, planned to testify that the girlfriend had recently told her that the boyfriend had hit her. The boyfriend’s attorney objects to the testimony of both witnesses.

How is the court likely to rule?

A The testimony of both the stranger and the best friend will be admissible.
B The testimony of neither the stranger nor the best friend will be admissible.
C The testimony of the best friend will be admissible and the testimony of the stranger will be inadmissible.
D The testimony of the stranger will be admissible and the testimony of the best friend will be inadmissible.

A

The correct answer is: The testimony of the stranger will be admissible and the testimony of the best friend will be inadmissible.

Discussion of correct answer: A general rule of evidence is that any out of court statement offered to prove the truth of the matter asserted is inadmissible hearsay, unless an exception applies. Here, the state is trying to prove that the boyfriend assaulted his girlfriend that night after they left the restaurant, and among the evidence it seeks to introduce are two pieces of testimony concerning what the girlfriend said. Both statements, the one made to the stranger and the one made to the best friend, were made out of court, and both are offered to prove the truth of the matter asserted, that the boyfriend hit his girlfriend. Unless an exception applies, neither will be admitted. The excited utterance exception will apply to the stranger’s testimony, but not to that of the best friend. Under the excited utterance exception, a statement that would otherwise be hearsay will be admissible if it was made under the stress of a startling or exciting event. The primary requirement is that the declarant was still under the influence of the emotional stress of the event when the statement was made. Here, the girlfriend was clearly under stress when she made the statement to the stranger, as she was running from her boyfriend, frantic. When she made the statement to her best friend, however, it was many days or even weeks later, after the stress of the incident had passed. Therefore, the statement to the best friend will not qualify as an excited utterance.

How well did you know this?
1
Not at all
2
3
4
5
Perfectly
14
Q

A restaurant patron sued a fast-food restaurant, alleging that she slipped and fell on a grease puddle on the floor that the restaurant had negligently failed to clean up within a reasonable time. As part of her case, to show the extent of damages, the restaurant patron offered the following testimony of her boyfriend: When he asked the restaurant patron to go sailing with him the day after the accident, she replied, “Sailing, no way! It will be a wonder if I can even stand up; my backside is killing me!” Opposing counsel objects to this testimony.

Should the trial court rule that the evidence is admissible or inadmissible?

A Inadmissible, because the declarant must be unavailable for a declaration of present bodily condition to be admitted.
B Inadmissible, because a recorded recollection must be in writing.
C Admissible, as an excited utterance.
D Admissible, as a declaration of present bodily condition.

A

The correct answer is:Admissible, as a declaration of present bodily condition.

Discussion of correct answer:This statement about the restaurant patron’s pain is within the hearsay exception for statements of present bodily condition. The present bodily condition exception is separate from the Federal Rule of Evidence exception for statements for the purpose of medical treatment, so it need not be to a medical provider or her agent to be admissible. The declarant’s availability is immaterial for both exceptions.

How well did you know this?
1
Not at all
2
3
4
5
Perfectly
15
Q

A woman called 911, telling the operator that her husband had hit her, threatened her children, and was now breaking objects in the living room. Once the husband saw her calling 911, he fled, and the caller told the operator where he went. The husband was arrested and charged with assault. At trial, the woman did not testify. Instead, the prosecution called the 911 operator to play the recording of what the woman said in her call. The husband’s attorney objected to the admission of the wife’s call to the police.

How should the trial court rule?

A The statement is inadmissible, because in criminal cases, all evidence presented against the defendant must be subject to cross-examination in front of the trier of fact.
B The statement is inadmissible, because it was testimonial in nature.
C The statement is admissible, as a statement made while under the stress of a startling event.
D The statement is admissible, because it was not testimonial in nature and qualifies as an excited utterance.

A

The correct answer is: The statement is admissible, because it was not testimonial in nature and qualifies as an excited utterance.

Discussion of correct answer: This call would be nontestimonial in nature (it is describing current events to help police deal with an emergency), and so everything the wife said, including the identity of the attacker, would be admissible in court. Therefore, even if she does not show up in court, the prosecutor can play the tape in order to get a conviction.

How well did you know this?
1
Not at all
2
3
4
5
Perfectly
16
Q

A college professor is on trial for the murder of her husband. The prosecution alleges that the professor paid one of her students to kill the victim. The student told his girlfriend the day after the husband’s death, “We’re rich! I just knocked off the old man and made $100,000 for five seconds’ work.” The student was a bit too exuberant, however, and a neighbor outside overheard every word. The student mysteriously disappeared before trial. Over the professor’s objection, the neighbor testified at trial as to the student’s remarks.

Why is the neighbor’s testimony regarding the student’s remarks admissible despite the professor’s objection?

A It is a statement with circumstantial guarantees of trustworthiness.
B It is a statement by an opposing party.
C It is a statement against penal interest.
D It is an excited utterance.

A

The correct answer is: It is a statement against penal interest.

Discussion of correct answer: If a declarant makes a statement against his penal interest and is unavailable for trial, the statement will qualify as a statement against interest and will be admitted as an exception to the hearsay rule. The facts indicate the student has mysteriously disappeared. The student’s statement also meets this exception’s criteria that it was against the student’s penal interest when he said it and that a reasonable person would not have made the statement if it were not true. Therefore, this is the best answer.

How well did you know this?
1
Not at all
2
3
4
5
Perfectly
17
Q

While driving her company vehicle near a pedestrian mall, a woman came upon the scene of a three-car accident. She was so busy gawking at the damaged vehicles that she failed to see one of the victims lying on the road in front of her car. She hit and ran over the victim, who survived and sued the woman’s company. The victim offers the testimony of a witness to the incident. Referring to the woman, the witness stated, “The driver of that car ran over the victim as he was lying on the ground awaiting an ambulance, and said ‘It is all my fault, I should have been paying more attention to my driving.’” Assume for this question that the woman is available to testify.

How should the trial judge rule on the admission of the testimony?

A Admissible, as a statement against interest.
B Admissible, as a present sense impression.
C Admissible, as a statement by a party-opponent.
D Inadmissible, as hearsay not within any recognized exception.

A

The correct answer is: Admissible, as a statement by a party-opponent.

Discussion of correct answer: A statement will not be barred from admission by the general rule against hearsay where the statement is offered against an opposing party and was made by the party’s agent or employee on a matter within the scope of that relationship and while it existed. Indeed, there is a substantial trend favoring the admission of statements relating to matters within the scope of the agency or employment. Therefore, this answer is correct, because the statement will be admitted as a vicarious statement by a party-opponent.

How well did you know this?
1
Not at all
2
3
4
5
Perfectly
18
Q

A woman sued her former employer for wrongful termination. During discovery, the woman’s attorney found out that the woman’s former supervisor, a key witness for the woman’s case, was terminally ill. The woman’s attorney deposed the supervisor. The employer’s attorney attended the deposition but did not cross-examine the supervisor.

If the supervisor dies prior to trial, is the supervisor’s deposition testimony admissible as substantive evidence?

A No, because the employer’s attorney did not cross-examine the supervisor.
B No, because the supervisor’s deposition is hearsay.
C Yes, because the supervisor’s deposition is former testimony.
D Yes, because the supervisor’s deposition was under the belief of imminent death.

A

The correct answer is: Yes, because the supervisor’s deposition is former testimony.

Discussion of correct answer: There is a hearsay exception for former testimony that applies when the declarant is unavailable to testify at trial. This exception requires that the testimony was given at a trial, hearing, or lawful deposition, and it is offered against a party who had an opportunity to develop the former testimony. The supervisor’s deposition is former testimony, and the supervisor is unavailable because she passed away. The employer’s attorney had the opportunity to cross-examine the supervisor at the deposition, and it is immaterial that the employer’s attorney chose not to do so. The supervisor’s deposition is therefore admissible substantively at trial as former testimony.

How well did you know this?
1
Not at all
2
3
4
5
Perfectly
19
Q

A brother and a sister were arguing about their parents’ estate, and it became a heated exchange where many long-simmering resentments exploded. The sister grabbed a lamp and threw it at her brother, and her brother then grabbed his sister and began choking her in anger and frustration. She collapsed and, realizing he had gone too far, he called 911. When the ambulance arrived, the sister was barely conscious. She was trying to speak, but the medic told her to stay calm and quiet until they could get her to the hospital or she risked death.

Moments later her neighbor came rushing in and dropped to the sister’s side, asking what happened and if she was ok. The sister mustered up every bit of energy she could and whispered to her neighbor, “Don’t worry about me, I need to tell someone that I saw my brother burn his house down last week.”

The sister was taken to the hospital and made a full recovery. Indeed, the brother’s house had burned down the week before, and he had submitted a claim with his insurance company. When the neighbor told the police about the sister’s statement, the prosecutor filed an arson charge against the brother.

At the arson trial, will the sister’s statement be admissible against the brother?

A No, because her statement does not concern the cause of her belief of imminent death.
B No, because she did not die.
C Yes, because she made the statement believing her death was imminent.
D Yes, because it is being offered in a criminal action.

A

The correct answer is: No, because her statement does not concern the cause of her belief of imminent death.

Discussion of correct answer: In general, a statement made out of court that is offered to prove the truth of the matter asserted is considered hearsay and will be inadmissible. There are certain exceptions, however, where testimony that is considered hearsay will be admitted. One such exception is a statement made under a belief that death is imminent. If a declarant makes an out of court statement under such a belief, and the statement is about the cause or circumstances leading to that belief, it will be admitted in a homicide or civil case if the declarant is unavailable to testify. Here, the state is seeking to admit a statement made by the sister out of court to prove the truth of the matter asserted, that the brother burned down his house. Her testimony is therefore inadmissible hearsay unless an exception applies. Although she was under a belief that her death was imminent, her statement made under that belief was not about the cause or circumstances surrounding her present condition. She made it clear that she was speaking about another incident entirely. Therefore, her statement will not qualify for this exception and will be inadmissible.

How well did you know this?
1
Not at all
2
3
4
5
Perfectly
20
Q

At a defendant’s trial for intoxication manslaughter, which of the following is admissible if offered to show she was intoxicated?

A The defendant’s bar receipt which was found in her car.
B The defendant’s statement to a bystander that the defendant didn’t realize how many drinks she had consumed.
C Testimony from a witness that the witness told the defendant after the accident that the defendant smelled like alcohol.
D Testimony from the bartender that the bartender told the defendant that the defendant was too intoxicated to drive.

A

The correct answer is: The defendant’s statement to a bystander that the defendant didn’t realize how many drinks she had consumed.

Discussion of correct answer: FRE 801 defines what constitutes hearsay and what statements are not considered hearsay. This is slightly different from the exceptions to hearsay contained in FRE 803 and FRE 804 because non-hearsay statements don’t go through the initial step of being classified as hearsay and then the second step of satisfying an exemption. One statement that is defined as non-hearsay is a statement by a party opponent if the statement is being offered against a party and was made by that party. Here, the defendant made the statement to a witness about how many drinks she had consumed. If it is being offered to show that she was intoxicated, it would be being offered by the prosecution, to which she is the party opponent. Her statement would therefore be non-hearsay and admissible at trial.

How well did you know this?
1
Not at all
2
3
4
5
Perfectly
21
Q

A heartsick lover knew he was being foolish, but he just could not stop himself. The woman he was in love with was so beautiful, and she was so lonely, with her husband in prison. She had told the lover that her husband was extremely jealous. She said that her husband was in prison because he had shot her last lover when he learned of the affair. About four months after he started seeing the woman, the lover received a telephone call from her. She seemed distraught, and cried, “My husband escaped from prison this evening, and he knows about us!” As expected, the husband called on the lover that evening. The lover shot the husband and was prosecuted for his murder. The lover claims self-defense. At trial, he attempts to testify to the woman’s statement that her husband was in prison because he had shot her last lover when he learned of the affair.

If the statement is offered to prove that the husband was in prison because he had shot the woman’s lover when he learned of the affair, should the statement be admitted?

a. Yes, under the state of mind exception.
b. Yes, as non-hearsay.
c. No, as hearsay not within any exception.
d. No, because it is self-serving.

A

c. No, as hearsay not within any exception.

How well did you know this?
1
Not at all
2
3
4
5
Perfectly
22
Q

A woman served for several years as the CEO of a successful Fortune 500 company until she was forced out by the board of directors. The company was involved in some environmental litigation against a group of public interest plaintiffs. Shortly after her resignation, and after she had moved to a different job, she wrote a letter to one of the plaintiffs that contained the following language, “We both know this litigation has dragged on too long. I’ve kept a secret for a long time, but I think it’s time you should know. Our company was responsible for dumping toxic waste in the protected wetland. We did everything we could to hide it from you, but it’s time you knew the truth. We did it, and we knew what we were doing.”

Is the woman’s letter admissible against the company in the environmental litigation?

A Yes, because it is a statement against interest.
B Yes, because it is a statement by a party-opponent.
C No, because the statement was not made at or near the time of the incident.
D No, because it is hearsay not within any exception.

A

The correct answer is: No, because it is hearsay not within any exception.

Discussion of correct answer: While it is tempting to claim that this is a vicarious statement by a party-opponent by an authorized agent, the problem with that theory is that the woman did not make the statement while she was employed by the company. In fact, she made it after her employment had ended. The statement might well be true, but she cannot bind the company by her statements the way that she could while she was still acting as CEO. The law permits agents of a company to bind the company with their statements while they are still an agent or employee. Once that relationship has ended, the person no longer has the ability to make a vicarious statement on behalf of the company. In this case, the woman was forced out as CEO and could have an incentive to be untruthful so as to hurt the company.

How well did you know this?
1
Not at all
2
3
4
5
Perfectly
23
Q

A plaintiff sued a defendant for injuries suffered in a fall on the sidewalk of the defendant’s home. The plaintiff’s complaint alleged that the walk was covered by a thick sheet of ice, which had been negligently left there for several days by the defendant. In his answer, the defendant set forth that the ice formed overnight and that the plaintiff fell before the defendant had a chance to remove it. During the trial, a physician, whose office was next door to the defendant’s home, testified that he saw the plaintiff fall and that the ice, which had been there for several days was at least two inches thick. On cross-examination, counsel for the defendant asked the physician the following question: “During your treatment of the plaintiff on the day in question, is it not true that he told you his fall resulted from a loss of equilibrium after he suffered from dizziness?”

Upon objection by the plaintiff’s attorney, how should the court rule on the physician’s testimony?

A Admissible, because it is reasonably pertinent to diagnosis or treatment.
B Admissible, because the physician had personal knowledge of the plaintiff’s injuries.
C Inadmissible, because the defendant’s counsel failed to lay a proper foundation.
D Inadmissible, because the physician’s testimony is not relevant to prove that the plaintiff’s alleged injuries are false or exaggerated.

A

The correct answer is: Admissible, because it is reasonably pertinent to diagnosis or treatment.

Discussion of correct answer: The Federal Rules of Evidence allows not only statements of past symptoms and medical history as exceptions under the hearsay rule, but also the cause or source of a patient’s past physical condition, insofar as it is reasonably pertinent to diagnosis or treatment. In this regard, the plaintiff’s statements to the physician would be admissible as reasonably pertinent to diagnosis and/or treatment regarding the cause or source of his injuries.

How well did you know this?
1
Not at all
2
3
4
5
Perfectly
24
Q

A husband and his wife recently put a down payment on a plot of land on which a developer planned to construct new homes. The contract between the husband and wife and the developer contained a “satisfaction guarantee” clause. Upon closing, the husband discovered that the house did not meet their specifications. The husband and wife brought suit against the developer, seeking specific performance. At trial, the developer’s attorney sought to question the wife as to whether she and her husband ever discussed any details of their soon-to-be home. The wife’s attorney objected to this question. Assuming that the state in which this trial is being held recognizes a privilege by one spouse not to testify against another spouse in a civil action, how should the judge rule on the objection?

a. Overruled, because the privilege of marital communications is lost when both the husband and the wife are parties to a suit.
b. Overruled, because the wife waived any objection to such testimony by taking the stand.
c. Sustained, because the question solicited information concerning private conversations between the husband and the wife.
d. Sustained, because the wife cannot be compelled to testify against the husband under any circumstance.

A

c. Sustained, because the question solicited information concerning private conversations between the husband and the wife.

How well did you know this?
1
Not at all
2
3
4
5
Perfectly
25
Q

A man was prosecuted for armed robbery of a local convenience store. The robber had been extremely unkempt, with long hair and a scruffy beard. The store clerk identified him in a photo lineup. At trial, the alleged robber wore a suit and tie, was clean-shaven, and had short hair. The prosecutor then showed the store clerk a group of photographs. The clerk testified that she had previously told the prosecutor that the third photograph was a picture of the robber. The third photograph was a picture of the suspect, with long hair and a beard.

Should the clerk’s testimony be admitted?

A No, because it violates the suspect’s confrontation rights.
B No, as hearsay not within any exception.
C Yes, as a prior identification.
D Yes, as a recorded recollection.

A

The correct answer is:Yes, as a prior identification.

Discussion of correct answer:The evidence is admissible as a prior identification. Federal Rule of Evidence 801(d)(1)(C) classifies a prior identification of a person, made after perceiving that person, as nonhearsay if the declarant (eyewitness) is available to testify and be cross-examined. Thus, the store clerk may testify that she earlier identified the defendant, and the prior identification may be used as substantive evidence.

How well did you know this?
1
Not at all
2
3
4
5
Perfectly
26
Q

A new husband and wife took a ferry ride on their honeymoon. As they were descending the observation deck, the husband slipped on some water on the stairs. The husband alleged that he sustained severe back injuries from the fall, and he sued the ferry owner. Although the ferry owner admitted that the husband was injured by slipping on the water, the ferry owner denied negligence and claimed that the husband was contributorily negligent. The wife died before the trial. At trial, the ferry owner called the ferry captain to testify that just before the husband fell, the captain heard someone call out, “Be careful, the stairs are wet.” The husband offered into evidence his wife’s deposition testimony that “I did not hear anyone warn my husband about the water on the stairs.”

Is the deposition testimony admissible?

A No, because the wife is not subject to cross-examination.
B No, because the testimony is hearsay not within any exception.
C Yes, as a dying declaration.
D Yes, as former testimony.

A

The correct answer is: Yes, as former testimony.

Discussion of correct answer: This is the best choice because Federal Rule of Evidence 804(b)(1) provides that testimony given by a witness in another hearing (including a hearing in a different proceeding) or taken in a deposition in the same (or different) proceeding is excepted from the hearsay rule if: (1) the former witness is unavailable in the present proceeding; and (2) the party against whom the former testimony is offered (or his predecessor in interest) had an opportunity to develop the former testimony by direct, redirect, or cross-examination and had a similar motive to develop the former testimony by cross-examination. Here, the wife is unavailable at trial and was subject to examination by the ferry owner at the deposition. Therefore, the former testimony exception applies.

How well did you know this?
1
Not at all
2
3
4
5
Perfectly
27
Q

A resident wanted to adopt a dog. Because of recent burglaries, the resident preferred to adopt a dog from a more aggressive breed to protect the apartment. Once he found a puppy, the resident took the dog to a veterinarian to make sure that it was healthy. While there, the veterinarian told the resident that this puppy was a member of a very aggressive dog breed. The veterinarian recounted a set of specific incidences involving dogs of this breed attacking children and mauling other animals. The resident still went forward and adopted the puppy.

A year later, the resident’s neighbor claimed the dog engaged in an unprovoked attack on the neighbor in the elevator, causing significant facial disfiguring. The resident argued that the injuries actually occurred when the neighbor attempted to break into the resident’s apartment and kill the dog in order to steal the resident’s valuables. The injured neighbor sued the resident. At trial, the veterinarian was called to testify about the dog’s health at the time of the adoption. On cross-examination, the neighbor elicited testimony from the veterinarian regarding his knowledge of prior incidences involving dogs of the same breed. This information was introduced to prove conduct in conformity therewith. The resident objected to the use of this evidence.

Should the judge sustain the resident’s objection and exclude this portion of the veterinarian’s testimony?

A Yes, this is an improper use of character evidence.
B Yes, this is protected by the physician-patient privilege.
C No, this is proper habit evidence.
D No, this is not being offered to prove the truth of the matter asserted.

A

The correct answer is: Yes, this is an improper use of character evidence.

Discussion of correct answer: There are general rules which apply to the admission of all evidence and special rules which apply to the admissibility of specific types of evidence such as evidence of subsequent remedial measures, habit evidence, and offers to compromise. Character evidence is also evidence which is subject to special rules governing admissibility. Generally, when we think of character evidence, we think of that character evidence as it pertains to the character of a human. However, character evidence can also be introduced as it pertains to animals in certain circumstances. Specific instances of animal behavior are generally admissible to prove conduct in conformity therewith. So, evidence that a specific animal has previously done a specific bad act can be admitted to show that the animal would be likely to commit that same bad act again. Here, if the veterinarian was being called to testify about an incident in which the resident’s dog has bad been violent in the past, then this type of character evidence would be perfectly admissible. However, under these facts, the veterinarian is being called to testify about specific incidences of bad animal acts which were committed by other animals. This is an improper manner of introducing evidence relating to the resident’s dog if being offered to prove conduct in conformity therewith. Therefore, it would be proper for the judge to sustain the objection and exclude the veterinarian’s testimony for this purpose.

How well did you know this?
1
Not at all
2
3
4
5
Perfectly
28
Q

A man was a convicted of sexual assault and required to register with the county of his residence within 30 days of his release from prison. After being paroled, the man rented an apartment, but failed to register. Three months later, during a routine traffic stop, the man was arrested for failing to register in violation of his parole.

At trial, the prosecutor called the chief clerk of the county in which the defendant’s apartment was located to testify. She proposed to testify that she did a diligent search of the country sex-offense registry and that the defendant’s name did not appear in the records. The defendant objected, claiming that her testimony was hearsay.

How should the court rule?

A The testimony is inadmissible, under the Confrontation Clause.
B The testimony is inadmissible, because the business record exception applies only to an actual entry in a business record.
C The testimony is admissible, as proof of the absence of an entry in a business record.
D The testimony is admissible, because the clerk has firsthand knowledge of the country records.

A

The correct answer is: The testimony is admissible, as proof of the absence of an entry in a business record.

Discussion of correct answer: In order to prove who has not registered as a sex offender, the clerk must testify to a document that lists all of the registered sex offenders. Because this list is an out-of-court statement being offered for its truth, a hearsay exception must exist for it to be admissible. Here, the prosecution is attempting to prove that the man is not on the list, and therefore not compliant with the law. As such, the absence of an entry in a business record is what is being proved. The elements are similar to the business record exception; however, it is being used to show what is not there, rather than what is there. Because the man’s absence from the list is what is being sought to be proved, this exception provides for the clerk’s testimony.

How well did you know this?
1
Not at all
2
3
4
5
Perfectly
29
Q

A man crashed into a woman’s car. Shortly after the accident, a police officer arrived at the scene, and the woman told the officer that the man had run the red light and swerved into her car. The police officer wrote her statement in the “Accident Report Form,” which he was required to complete after every accident. The woman sued for personal injuries sustained in the accident. On direct examination, her attorney called her to testify and asked her to recall the events of the night.

Is the woman’s testimony admissible?

A No, because a party is precluded from introducing their own out-of-court statements at a subsequent trial.
B No, because it is hearsay not within any exception.
C Yes, because the officer wrote down the statement as part of his official duties.
D Yes, because the woman has firsthand knowledge of the events of the night in question.

A

The correct answer is: Yes, because the woman has firsthand knowledge of the events of the night in question.

Discussion of correct answer: The woman is not being asked to recount her out-of-court statement, but rather, to testify as to what she remembered seeing on the night in question. As long as the witness has firsthand knowledge, she is a proper witness to the events, and can testify as to what she saw. Therefore, there is no hearsay problem.

How well did you know this?
1
Not at all
2
3
4
5
Perfectly
30
Q

The plaintiff attended a party at the defendant’s apartment. During the party, the area was hit by a fast-moving snowstorm that blanketed the ground with six inches of snow. When the plaintiff left the party, he slipped on the snow covering the front porch and broke his leg. The plaintiff sued the defendant for negligence in failing to warn him about the snow on the porch. At trial, the plaintiff called another guest at the party as a witness, who testified that shortly before the plaintiff left the apartment, another partygoer had gone outside to smoke a cigarette, and when he returned, he stated: “There is six inches of snow on the front porch.” He further testified that the defendant was present when this statement was made. The defendant’s attorney objected to this testimony.

Is the statement of the partygoer admissible?

A No, as hearsay not within an exception.
B No, unless the partygoer is unavailable to testify at trial.
C Yes, as a statement describing a condition that the partygoer just observed.
D Yes, as lay opinion.

A

The correct answer is: Yes, as a statement describing a condition that the partygoer just observed.

Discussion of correct answer: FRE 803(1) provides an exception to the hearsay rule for statements describing a condition made immediately after observing it–the present sense impression exception. Here the witness is testifying about another person’s statement that was made out-of-court and now being offered in court for its truth–to show that there was snow on the porch. It is a hearsay statement, but is admissible under the present sense impression exception. Some students may recognize another theory of admissibility. Here, the statement of the partygoer could be viewed as a nonhearsay statement of warning. In other words, the statement is being offered not for its truth, but to show that the defendant was aware of the danger of the snow on the porch. Because it is not offered for its truth, it is nonhearsay. However, there is no answer choice that reflects this theory, so this is the best answer choice.

How well did you know this?
1
Not at all
2
3
4
5
Perfectly
31
Q

A chef was involved in a two-car collision and sued the other driver, a cook. At the trial, the chef testified that the cook failed to stop at a stop sign and hit her, so the accident was not her fault. The cook called a witness who testified that she has known the cook for many years, and she observed that he always completely stopped at every stop sign.

Is the witness’s testimony admissible?

A Yes, because it tends to establish a pertinent habit of the cook’s while driving.
B Yes, because it is evidence of the cook’s cautious character behind the wheel.
C No, because the cook’s reputation for careful driving can only be proved by opinion or reputation.
D No, because the cook’s prior conduct cannot be used to prove he is a careful driver.

A

The correct answer is: Yes, because it tends to establish a pertinent habit of the cook’s while driving.

Discussion of correct answer: Under Federal Rule of Evidence 406, evidence of a person’s habit can be used to show that a person acted in a similar matter on a particular occasion. Habit or routine practice may be proved by testimony in the form of an opinion or by specific instances of conduct sufficient in number to warrant a finding that the habit existed or that the practice was routine. Always stopping at stop signs is the kind of automatic, invariable behavior for which this type of testimony may be admitted. The witness may therefore testify that the driver always stopped at stop signs, according to her observation.

How well did you know this?
1
Not at all
2
3
4
5
Perfectly
32
Q

The plaintiff is suing the defendant for damages as the result of an automobile accident. The plaintiff offers the testimony of a police officer who arrived at the scene of the accident one hour after it occurred. There he met a bystander who claimed she saw the defendant’s car drive through a red light and strike the plaintiff’s car. The bystander wrote her statement on a piece of paper and handed it to the police officer. The police officer retained possession of the note, which is now being offered in evidence at trial. Defense counsel objects.

Is the note admissible?

A Yes, as a prior consistent statement.
B Yes, as past recollection recorded.
C No, under the best evidence rule.
D No, as hearsay not within an exception.

A

The correct answer is: No, as hearsay not within an exception.

Discussion of correct answer: The note is an out-of-court statement of the bystander which is being offered in court by the plaintiff for its truth (i.e., that the defendant caused the accident). It is, therefore, hearsay. There is no exception that applies here, although some might argue that it may qualify as a present sense impression. However, because the statement was written at least one hour after the accident, it would not meet the requirement that it be made contemporaneously with the observation or immediately thereafter.

How well did you know this?
1
Not at all
2
3
4
5
Perfectly
33
Q

A husband and wife were experiencing marital problems. The husband suspected that his wife was having an affair with his best friend. One evening, the husband came home to discover his best friend and his wife sitting at the kitchen table having a cup of coffee. Their clothing looked messy and they both appeared to blush when he asked what they had been doing. The husband yelled, “You have been cheating on me with him haven’t you? How could you do this to me?” The wife made no reply. A week later, the husband filed for divorce. At the divorce trial, during the husband’s testimony, his attorney attempted to introduce into evidence that fact that the wife was silent in response to the husband’s accusations. The wife’s attorney objects.

How should the court rule?

A The court should overrule the objection, because the husband is on the stand and is subject to cross-examination.
B The court should overrule the objection, because the wife’s silence constitutes an adoptive statement by silence.
C The court should sustain the objection, because the husband’s statement is hearsay not within any exception.
D The court should sustain the objection, because the statement is in the husband’s self interest.

A

The correct answer is: The court should overrule the objection, because the wife’s silence constitutes an adoptive statement by silence.

Discussion of correct answer: Rule 801(d)(2) states that statements by acquiescence or silence can be offered into evidence and are not hearsay. Under this doctrine, when an accusatory statement is made in the presence of an individual, who understands and has an opportunity to deny it but remains silent, the statement and failure to deny it are admissible as an adoptive statement by silence. The basis for this doctrine is that an innocent person would speak up and defend themselves under the circumstances.

How well did you know this?
1
Not at all
2
3
4
5
Perfectly
34
Q

A plaintiff brought a personal injury action against a delivery truck company for damages arising from a car accident in which the plaintiff’s car collided with a delivery truck driven by one of the company’s drivers. During his lunch break, the delivery truck driver had driven the delivery truck home, where he consumed several alcoholic drinks, and then had taken the rest of the day off to attend a baseball game. The collision occurred in the parking lot of the baseball stadium.

In his complaint, the plaintiff alleged that the delivery truck company’s personnel manager had been negligent in approving the hiring of the delivery truck driver, who was an alcoholic, and that the manager reasonably should have known that the driver’s alcoholism made him a serious risk to others while operating a motor vehicle. At trial, the plaintiff called the driver’s former wife to testify that she had lived with the driver for the six months prior to the accident and that on three separate occasions, she had observed the driver operate a motor vehicle while in a severely intoxicated state. The delivery truck company objected to the introduction of the former wife’s testimony. Should the court admit the former wife’s testimony over the truck company’s objection?

a. Yes, because the driver’s character is a material issue in the litigation.
b. Yes, because the driver is not a defendant in this case.
c. No, because evidence of specific instances of misconduct may only be inquired into on cross-examination and may not be proven by extrinsic evidence.
d. No, unless the driver had already testified that he has never operated a motor vehicle while intoxicated.

A

a. Yes, because the driver’s character is a material issue in the litigation.

How well did you know this?
1
Not at all
2
3
4
5
Perfectly
35
Q

A motorcyclist was involved in a collision with a pickup truck on the highway late one evening. The responding police officer arrested the motorcyclist for driving under the influence of alcohol and failure to maintain a lane. A blood sample was taken from the motorcyclist at the hospital before he was transported by police to the jail. At trial, the defense presented a scientist as an expert witness to testify about the inaccuracies in blood sample testing when blood was collected in the manner that the police used in this case. The expert witness also testified that the motorcyclist likely was not under the influence of alcohol to the extent claimed by the prosecution. One year prior to testifying in this case, the expert witness had left his previous place of employment where he had worked for the government for fifteen continuous years. On cross-examination, the prosecution attempted to introduce a news article published two years earlier in which the expert is quoted as saying that that accuracy of the method of blood sample testing used in all criminal prosecutions was irrefutable. What would be a proper basis for the introduction of this article?

a. The article contains prior inconsistent statements that may be used to prove the truth of the matter asserted.
b. The article contains prior sworn inconsistent statements that may be used as substantive evidence.
c. The article contains prior inconsistent statements that may be used for impeachment purposes.
d. The article contains a statement that may be used as substantive evidence.

A

c. The article contains prior inconsistent statements that may be used for impeachment purposes.

How well did you know this?
1
Not at all
2
3
4
5
Perfectly
36
Q

A real estate company occasionally hired temporary workers from a local agency. During an especially busy month, the real estate company needed an accountant to assist in its financial data operations. The real estate company contacted the agency’s staffing agent and requested an accountant for temporary employment. The real estate company and the staffing agent then entered into a contract in which the staffing agent promised to provide an accountant “who was honest and trustworthy.” A short time later, the staffing agent sent over a woman who was placed in the accounting position.

The woman worked at the real estate company for one week, until she was fired for stealing money. The real estate company then brought suit against the agency for negligent hiring. During the trial, the agency denied having any knowledge that the woman they sent over was dishonest and untrustworthy. Thereupon, the real estate company proffered evidence to show that two months before the agency referred the woman to the real estate company, she had been terminated from a previous temporary employment job for theft. Upon objection by the agency, is the proffered evidence admissible?

a. Yes, because specific instances of conduct may be admitted for the limited purpose of impeachment.
b. Yes, as proof of the woman’s character trait for dishonesty.
c. No, because the agent’s character is in issue, not the women, because the agent was the one who referred her.
d. No, because specific instances of conduct are an improper means of proving character.

A

b. Yes, as proof of the woman’s character trait for dishonesty.

How well did you know this?
1
Not at all
2
3
4
5
Perfectly
37
Q

A man and a woman were involved in a car accident that occurred when the man abruptly switched lanes in front of the woman without signaling. Unable to stop, the woman hit the man’s car into a median. Immediately after the accident, as the two drivers emerged from their vehicles, a pedestrian, obviously emotional and upset from having just observed an accident, ran up to the man and shouted, “You careless driver, you! Didn’t you ever learn that you’re supposed to use a turn signal when you switch lanes?” The pedestrian was never identified and is not present at trial.

How should the court rule on the pedestrian’s statement?

A Admissible, as nonhearsay.
B Admissible, even though it is hearsay.
C Admissible, under the excited utterance exception.
D Inadmissible, because the bystander cannot be identified and is not present at trial.

A

The correct answer is: Admissible, under the excited utterance exception.

Discussion of correct answer: Under FRE 803(2), a statement relating to a startling event or condition made while the declarant was under the stress of excitement caused by the event or condition is not excluded by the hearsay rule. The theory behind the excited utterance exception is simply that circumstances may produce a condition of excitement that temporarily stills the capacity of reflection and, thus, produces utterances free of conscious fabrication. Spontaneity is the key factor in determining whether the statement was, indeed, made without any conscious reflection.

How well did you know this?
1
Not at all
2
3
4
5
Perfectly
38
Q

A man was on trial for kidnapping a woman and holding her prisoner for nearly a week. Photographs of the man’s home were introduced into evidence, including photos of the attic where the woman was kept. During the week she was held captive by the man, the woman never saw his face or any part of the house other than the attic. She was, however, able to keep a small notebook that she wrote down observations in. On the third day, she made an entry stating that the house smelled like something was burning and that she could feel heat and see smoke coming through the door on the floor of the attic. The prosecutor wished to offer the notes as evidence because a fire was reported at the man’s house the same day the woman made the entry. The woman was available to testify at trial about the abduction and the entries in her notebook.

Are the woman’s notes admissible to show that there had been a house-fire that day?

A No, because the matter asserted is that there was something burning and the statement is being offered to prove that the house was burning.
B No, because the woman is not unavailable for trial.
C Yes, because the notes are the woman’s recorded recollection.
D Yes, because the notes describe an event that the woman observed.

A

The correct answer is: Yes, because the notes describe an event that the woman observed.

Discussion of correct answer: One exception to the rule against hearsay is the exception for present sense impressions under FRE 803. A statement qualifies under the exception if it is a statement that explains or describes an event or condition made while or immediately after it was perceived. In this case, the woman wrote down her observation of the smell, heat and smoke as she observed it. Thus, answer (D) is correct.

How well did you know this?
1
Not at all
2
3
4
5
Perfectly
39
Q

The driver of a minivan ran a red light and slammed into the passenger side of another car. The passenger, a teenage girl, was injured. Her parents later brought a civil suit against the driver of the minivan. The day before the trial, the teenage girl turned 18 and hopped a flight to Africa for a two-year mission with the Peace Corps. At trial, the parents alleged that their daughter had sustained head and neck injuries in the car accident and has since been unable to move her neck normally. For his defense, the minivan driver brought in an as a witness the ambulance medic who responded at the scene. The medic planned to testify that at the scene of the accident he asked the girl if she could move her neck freely, and that she had nodded yes. The attorney for the parents objected, stating that such testimony was inadmissible hearsay evidence.

How is the court likely to rule on the admissibility of the medic’s testimony?

A Admissible, because the teenage girl is unavailable to appear personally in court.
B Admissible, because the evidence is not being offered to prove the truth of the matter asserted.
C Inadmissible, because the evidence is being offered to prove the truth of the matter asserted.
D Inadmissible, because there is no evidence that the teenage girl made the assertion under a belief that death was imminent.

A

The correct answer is: Admissible, because the evidence is not being offered to prove the truth of the matter asserted.

Discussion of correct answer: The general rule on hearsay is that an out of court statement is not admissible to prove the truth of the matter asserted. At issue here is whether the teenage girl was able to move her neck freely following the accident. The defense seeks to admit evidence that she answered the medic’s question by nodding her head immediately after the accident, proving that she was in fact able to move her neck. Although the girl’s nodding is an out-of-court nonverbal assertion answering the medic’s question, and it is being offered to prove that she could indeed move her neck, it is not hearsay. Her affirmative response to the medic’s question is not what the defense is attempting to offer into evidence. The medic is simply testifying that he saw with his own eyes that she could physically move her neck.

How well did you know this?
1
Not at all
2
3
4
5
Perfectly
40
Q

Three men and a woman robbed a bar and its patrons on December 16 at 5:00 p.m. The defendant, alleged to be one of the robbers, called a defense witness to testified that she drove to the defendant’s home at 4:00 p.m. on December 16 and picked him up for a tennis game, which lasted until 5:30 p.m. The prosecutor asked on cross-examination, “What is your relationship to the defendant?” The defense counsel objected. How should the court rule?

a. Overruled, because the question is directed at discovering possible bias in the witness.
b. Overruled, because the question attacks the witness’s truth and veracity.
c. Sustained, because the question seeks to elicit irrelevant information.
d. Sustained, because the answer to the question would create prejudice that would outweigh its probative value.

A

a. Overruled, because the question is directed at discovering possible bias in the witness.

How well did you know this?
1
Not at all
2
3
4
5
Perfectly
41
Q

A local TV news anchor aired a show in which she stated that the mayor was “a corrupt politician who manages to grab a surprisingly large amount of bribe money with his small hands.” The mayor was the subject of a recall effort that was ultimately successful, and he sued the TV station for defamation. The TV station defended on the grounds of truth. The mayor took the stand and testified that he was an honest politician. The station called a witness to testify that a year prior to the TV show in question, the mayor took a bribe in connection with a rezoning project. The mayor’s lawyer objects. Should the court allow the testimony to be admitted into evidence?

a. No, because it is inadmissible character evidence.
b. No, because the prosecutor may not introduce evidence of character unless the defendant puts his character in issue.
c. Yes, as relevant evidence of the mayor’s character as a corrupt politician.
d. Yes, as relevant evidence of character for truthfulness.

A

c. Yes, as relevant evidence of the mayor’s character as a corrupt politician.

How well did you know this?
1
Not at all
2
3
4
5
Perfectly
42
Q

A man sued his company claiming that he was denied his yearly bonus. The CEO claimed that the man was not a bonused employee. To prove this fact, he wished to introduce a record of all bonused employees. The CEO called the secretary who was in charge of payroll to testify to this record. The man objected claiming the report was hearsay.

How should the court rule?

a. The testimony is inadmissible under the Confrontation Clause.
b. The testimony is inadmissible because the business record exception applies only to an actual entry in a business record.
c. The testimony is admissible as proof of the absence of entry in business records.
d. The testimony is admissible as habit evidence provided there is corroborating evidence.

A

c. The testimony is admissible as proof of the absence of entry in business records.

How well did you know this?
1
Not at all
2
3
4
5
Perfectly
43
Q

A defendant is on trial for murder. The defendant allegedly stabbed the victim with a knife. The defendant claims self-defense and offers the testimony of the victim’s wife that her husband had a reputation in the community for having a violent temper. On rebuttal, the prosecution offers the testimony of the defendant’s former business partner that the defendant, in his opinion, is a violent person. The defendant objects, claiming the testimony is improper character evidence. How should the court rule on the proposed testimony?

a. It is inadmissible, because the defendant did not first offer evidence of his character for nonviolence.
b. It is inadmissible, because character evidence is inadmissible to prove a criminal disposition or propensity for violent behavior.
c. It is admissible, because the defendant opened the door to his character by first attacking the victim’s character for violence.
d. It is admissible, because the defendant’s character for violence is always in issue on a claim of self-defense.

A

c. It is admissible, because the defendant opened the door to his character by first attacking the victim’s character for violence.

How well did you know this?
1
Not at all
2
3
4
5
Perfectly
44
Q

The defendant was shopping in a convenience store. He decided to steal a package of batteries off the shelf. He picked up a package and placed it in his pocket. He was observed placing something in his pocket by the security guard, but the security guard could not see exactly what item was taken. The defendant then ran out of the store with the security guard in hot pursuit. Just as the security guard tackled the defendant, he threw the batteries into a dumpster. The security guard searched the defendant but found nothing. The defendant was charged with larceny. At trial, the prosecutor called the manager of the store as a witness. The manager was asked if he was able to determine what the defendant stole from the store. The manager said that although the security guard was unable to see what was taken, the manager was able to figure it out on his own. He testified that he reviewed the inventory records kept by the store, which had previously been entered into evidence, and was able to determine that one package of batteries was missing. The defendant’s attorney objected to this testimony.

Is the manager’s testimony admissible?

A No, because the manager did not have firsthand knowledge of what was stolen.
B No, because the inventory records are hearsay.
C Yes, because the inventory records are past recollection recorded.
D Yes, because the inventory records were made and kept by the business for just such a circumstance.

A

The correct answer is: Yes, because the inventory records were made and kept by the business for just such a circumstance.

Discussion of correct answer: FRE 803(6) makes an exception to the hearsay rule for records made and kept in the ordinary course of business. Here, the inventory records were made and kept by the store to track the items on the store’s shelves. They are classic business records.

How well did you know this?
1
Not at all
2
3
4
5
Perfectly
45
Q

A plaintiff sued a defendant for defamation after the defendant told the plaintiff’s co-workers that she had stolen money from the office petty cash box. The defendant testified in his own defense, but his attorney only asked him one question: “Did you tell the plaintiff’s co-workers that she had stolen petty cash?” The defendant responded: “No.” On cross-examination, the plaintiff’s attorney asked: “Isn’t it true that you were convicted of perjury nine years ago?” The defendant’s attorney objects. Should the court sustain the objection?

a. No, because counsel may inquire into matters bearing upon the credibility of the witness during cross-examination.
b. No, because the commission of a prior crime can be used to establish motive.
c. Yes, because of the highly prejudicial nature of the evidence.
d. Yes, because the question exceeds the scope of direct examination.

A

a. No, because counsel may inquire into matters bearing upon the credibility of the witness during cross-examination.

How well did you know this?
1
Not at all
2
3
4
5
Perfectly
46
Q

After being passed over for a promotion, an aeronautic engineer became a whistleblower. He contacted a government agency to state that the company for which he worked was submitting false safety reports on newly built aircraft. When the company learned that the engineer was leaking this information to the government agency, he was fired from his job. Afterward, the engineer sued the company for wrongful termination of employment. During the discovery stage of litigation, the engineer was deposed by the company’s attorney. In his deposition, the engineer stated that the company submitted false safety reports to the government agency to cover up structural defects in its aircraft. A pilot was injured when one of the company’s airplanes he was piloting crashed. The pilot asserted a strict products liability tort against the company. At trial, the pilot sought to introduce into evidence portions of the engineer’s deposition from his wrongful termination lawsuit against the company. Assume that the engineer is unavailable to testify at trial.

Upon objection by the company, how should the trial court judge rule on the engineer’s deposition?

A Admissible, as former testimony.
B Admissible, as a vicarious statement.
C Inadmissible, as hearsay not within any recognized exception.
D Inadmissible, because the company did not have the opportunity to cross-examine the engineer on the liability issue for which the statement is now being offered.

A

The correct answer is: Admissible, as former testimony.

Discussion of correct answer: Usually, former testimony questions will involve the same parties where each had the opportunity to interrogate the witness at the earlier trial. In this question, we have a plaintiff (who was not party to the first suit) seeking to introduce former testimony against the same defendant. According to Lilly, “using the testimony against the same defendant, which was a party to both suits and had an opportunity at the first trial to interrogate the witness, appears to fit comfortably within the exception.” Evidence, p. 286. On the other hand, Lilly points out that if a plaintiff seeks to use all or part of the former testimony against a new defendant (who did not have an opportunity to interrogate the witness), then the testimony should be excluded. Here, because the testimony is being used against the same defendant, it is admissible.

How well did you know this?
1
Not at all
2
3
4
5
Perfectly
47
Q

A social activist was appalled at the actions of a foreign multinational corporation in Asia. She took part in a large protest in front of the corporation’s American headquarters. Police moved to break up the protest, and the activist scuffled with police. She was arrested by an officer whom she had encountered before at rallies and demonstrations and whom she knew had a reputation for violence. The activist was charged with non-deadly assault against a police officer in the line of duty, a misdemeanor. During pretrial discovery, the activist learned that, during the officer’s eight years with the department, a local coalition against police brutality had twice protested the officer’s conduct, leading to his receiving official reprimands. The activist claims in her case-in-chief that she acted in self-defense and seeks to present a witness, who is a member of the coalition, to testify as to the officer’s reputation for violence. The prosecutor objects, arguing that the testimony is inadmissible character evidence.

Will the prosecutor’s objection succeed?

A No, because the officer’s reputation for violence is not an ultimate issue in the case.
B No, because evidence of the officer’s character may be admitted in this situation.
C Yes, because character evidence is never admissible in a criminal case, except for purposes of impeachment.
D Yes, because character traits may be proven only by evidence of specific acts.

A

The correct answer is: No, because evidence of the officer’s character may be admitted in this situation.

Discussion of correct answer: Most courts admit evidence of a victim’s character when it is relevant to an asserted defense, as self-defense is here. Some limit this to evidence of reputation, and a growing number allow opinion testimony as well. In addition, FRE 404(a)(2) provides that in a criminal case, evidence of a pertinent trait of character of the alleged victim of the crime offered by an accused is admissible. Here, the activist is claiming that she acted in self-defense, so the officer’s character and reputation for violence are relevant to the question of whether the activist acted in the reasonable belief that she needed to defend herself against the officer. Therefore, the activist may prove the officer’s character for violence with testimony about his specific evidence of his behavior, including evidence of the two reprimands for use of excessive force.

How well did you know this?
1
Not at all
2
3
4
5
Perfectly
48
Q

A truck driver for a local beer distributor was pulling onto the freeway when he collided with a car driven by a bank teller. The bank teller and her passengers were seriously injured in the accident, and they filed suit against the truck driver, alleging that he failed to yield as he entered the freeway from the entrance ramp. They also alleged that he had exceeded by five hours the company’s 12-hour limit for drivers per day, and that he should not have been driving at all, as he was taking medication that caused dizziness and fatigue. The truck driver, in turn, claimed that the bank teller was speeding and had failed to make any attempt to avoid the accident even though the two lanes to the left of the entrance ramp were wide open. The truck driver’s attorney seeks to introduce the testimony of the state trooper who first arrived on the scene that, when he approached the bank teller’s car immediately following the accident, she stated over and over, “I should have slowed down or moved over.” The bank teller objects.

Is the trooper’s statement admissible?

A Yes, because it was overheard by an officer during his investigation.
B Yes, because it is a statement by the bank teller.
C No, because it does not admit any wrongdoing that is clearly a violation of the motor vehicle law.
D No, because it is hearsay.

A

The correct answer is: Yes, because it is a statement by the bank teller.

Discussion of correct answer: Under Federal Rule of Evidence 801, a statement by a party-opponent that is offered against that party is not hearsay. Here, the bank teller’s statement that she should have slowed down or moved over constitutes an opposing party’s statement, and will therefore be admissible against her at trial.

How well did you know this?
1
Not at all
2
3
4
5
Perfectly
49
Q

The defendant was tried in state court for robbery of a bank. After he was acquitted, the defendant celebrated with friends at a local bar. During the celebration, the defendant was overheard by an off-duty FBI agent telling his friends that he did commit the robbery, but had a really good defense lawyer. The FBI agent then reported what he heard to a federal prosecutor, who sought and obtained an indictment against the defendant for a federal charge of bank robbery. At trial, the defendant invoked his Fifth Amendment privilege against self-incrimination, which the judge permitted. The prosecutor then offered the testimony of the FBI agent, who sought to testify about what the defendant said after his acquittal. Assume that the defendant’s motion for dismissal on double jeopardy grounds was denied.

Is the FBI agent’s testimony admissible?

A No, because it is a violation of the defendant’s right against self-incrimination.
B No, because it is hearsay not within an exception.
C Yes, as a statement against interest.
D Yes, as an opposing party’s statement.

A

The correct answer is: Yes, as an opposing party’s statement.

Discussion of correct answer: Students must be aware that the bar examiners have a preference for the nonhearsay answer choice over the exception-to-the-hearsay-rule answer choice. If the statement constitutes nonhearsay, then it is obviated from needing a hearsay exception for admission. In other words, in analyzing the bases for a statement’s admission that appears to have a hearsay problem, first see if it constitutes nonhearsay. If not, and the statement is hearsay, then look to see if a relevant exception applies. Here, the defendant’s statement is clearly against his penal interest under Rule 804, and the defendant is unavailable to testify as required by Rule 804 by virtue of invoking his Fifth Amendment right against self-incrimination. However, his statement is also nonhearsay–it is a statement by a party offered by the opponent party, and the Federal Rules of Evidence define opposing party’s statements as nonhearsay. The bar examiners prefer this answer choice to a statement against interest because it presents a nonhearsay reason rather than a hearsay exception.

How well did you know this?
1
Not at all
2
3
4
5
Perfectly
50
Q

A social activist was appalled at the actions of a foreign multinational corporation in Asia. She took part in a large protest in front of the corporation’s American headquarters. Police moved to break up the protest, and the activist scuffled with police. She was arrested by an officer whom she had encountered before at rallies and demonstrations and whom she knew had a reputation for violence. The activist was charged with non-deadly assault against a police officer in the line of duty, a misdemeanor. During pretrial discovery, the activist learned that, during the officer’s eight years with the department, a local coalition against police brutality had twice protested the officer’s conduct, leading to his receiving official reprimands. The activist claims in her case-in-chief that she acted in self-defense and seeks to present a witness, who is a member of the coalition, to testify as to the officer’s reputation for violence. The prosecutor objects, arguing that the testimony is inadmissible character evidence. Will the prosecutor’s objection succeed?

a. No, because the officer’s reputation for violence is not an ultimate issue in the case.
b. No, because evidence of the officer’s character may be admitted in this situation.
c. Yes, because character evidence is never admissible in a criminal case, except for purposes of impeachment.
d. Yes, because character traits may be proved only by evidence of specific acts.

A

b. No, because evidence of the officer’s character may be admitted in this situation.

How well did you know this?
1
Not at all
2
3
4
5
Perfectly
51
Q

Two police officers, a sergeant and a lieutenant, were charged with robbery and felony murder. The sergeant’s lawyer moved to have the two officers tried separately, and the court granted the motion. The sergeant’s trial was held first, and he was acquitted. At the lieutenant’s trial, his lawyer called the sergeant’s wife as a witness. The sergeant’s wife testified that after the sergeant was acquitted, the sergeant told her that he had committed the robbery because he wanted to buy her a new ring and that he felt bad because the lieutenant was not involved.

Should the wife’s testimony be admitted?

A No, because the sergeant is estopped from contradicting the jury’s acquittal verdict.
B No, because it is hearsay not within any exception.
C Yes, because it is a statement by a co-conspirator.
D Yes, because it is a statement against a penal interest.

A

The correct answer is: No, because it is hearsay not within any exception.

Discussion of correct answer: Pursuant to Federal Rule of Evidence 801(c), hearsay is an out-of-court statement offered to prove the truth of the matter asserted therein. Some out-of-court statements are admissible under exceptions to the rule excluding hearsay evidence. Furthermore, a statement is not hearsay if it is offered to prove something other than the truth of the matter asserted in the statement. Here, the wife’s testimony is being used to prove the truth of the matter asserted therein–that is, the lieutenant did not participate in the crime. Therefore, the testimony is hearsay. Because the statement does not fall within any of the exceptions to the hearsay rule, it cannot be admitted.

How well did you know this?
1
Not at all
2
3
4
5
Perfectly
52
Q

A defendant was on trial for attempted murder. The alleged victim was called by the prosecution to testify. During her testimony, the victim recounted the incident and described how the defendant savagely beat her with a baseball bat. She was not asked by the prosecution whether she made any statements during the attack. After the victim was excused and left the witness stand, the prosecution called another witness to testify. The witness proposes to testify that when the beating stopped, the victim screamed, “I’m dying. Don’t let the defendant get away with this.”

Upon objection by the defendant’s attorney, how will the court rule on the witness’s proffered testimony?

A Admissible, as an excited utterance.
B Admissible, as a dying declaration.
C Inadmissible, as hearsay not within any recognized exception.
D Inadmissible, because the victim was not questioned about the statement before being excused.

A

The correct answer is: Admissible, as an excited utterance.

Discussion of correct answer: Under FRE 803(2), an excited utterance is defined as a statement relating to a startling event or condition made while the declarant was under the stress of excitement caused by the event or condition. The witness’s testimony of the victim’s statement, “I’m dying. Don’t let the defendant get away with this,” related to the savage beating (the startling event) and was made when the beating stopped (while the victim was still under the stress of excitement). The testimony will be admissible substantively as an excited utterance.

How well did you know this?
1
Not at all
2
3
4
5
Perfectly
53
Q

The plaintiff sued a legal journal for libel based on an article which implied that he had cheated the IRS by not paying his taxes. At trial, the plaintiff denied any wrongdoing. An accountant was then called by the defense to testify that five years earlier, the plaintiff had improperly distorted the value of certain investments in an effort to lower his taxable income. Should the accountant’s testimony be admitted?

a. Yes, to challenge the plaintiff’s credibility.
b. Yes, as substantive evidence of the plaintiff’s character.
c. No, because it is improper character evidence.
d. No, because the testimony must be corroborated.

A

b. Yes, as substantive evidence of the plaintiff’s character.

How well did you know this?
1
Not at all
2
3
4
5
Perfectly
54
Q

A defendant was on trial for drug distribution. The arresting officer was called by the prosecution to describe in detail the events that led to the defendant’s arrest. The officer described the night in question, where he was, and what he observed. He was asked to provide a list of what he confiscated from the defendant after he was arrested. The officer testified that the defendant has a large amount of money and many small packets of drugs in several pockets throughout his clothing. When asked specifically about the number of drugs packets, the officer was unable to recall. The officer was then shown his police report. The officer quickly flipped to the third page of the report, then looked at the prosecutor. The prosecutor asked if the officer was now able to remember how many bags of drugs were taken from the defendant. The officer indicated that he was, but before he could answer the question, the defense attorney objected to the officer’s testimony.

How should the court rule?

A Sustain the objection, because the officer lacks a present memory of the incident.
B Sustain the objection and require the report to be admitted into evidence, because the jury is just as capable as reading the report as the officer.
C Overrule the objection, because the officer’s memory was refreshed.
D Overrule the objection, because the report is admissible as a past recollection recorded.

A

The correct answer is: Overrule the objection, because the officer’s memory was refreshed.

Discussion of correct answer: The prosecutor is refreshing the officer’s recollection. To do so, he can use anything available, because it is not being admitted as an exhibit, and so it is not used substantively. If the attempt to refresh fails, then extrinsic evidence is required, at which point past recollection recorded would be a valid hearsay exception. However, because the officer’s memory was refreshed, the judge should overrule the defense attorney’s objection.

How well did you know this?
1
Not at all
2
3
4
5
Perfectly
55
Q

A girlfriend and boyfriend lived together and had a tumultuous relationship. One night, the couple got in a fight. The next morning, the girlfriend realized that the boyfriend had moved out. The girlfriend also noticed that her valuable diamond necklace that she had inherited from her grandmother was gone. The girlfriend sued the boyfriend for conversion of the diamond necklace. At trial, to prove the location of the necklace before it went missing, the girlfriend testified that she wore the necklace everyday, and she always took the necklace off before she went to bed and placed it in the same place in her jewelry box. No other witness can attest to the girlfriend’s testimony. The boyfriend’s attorney objected to the girlfriend’s testimony regarding the necklace.

How should the judge rule on the objection?

A Overrule it, because the girlfriend’s testimony is relevant to whether the boyfriend converted the necklace.
B Overrule it, because the girlfriend’s testimony is admissible habit evidence.
C Sustain it, because the girlfriend’s testimony cannot be corroborated.
D Sustain it, because the girlfriend’s testimony is inadmissible character evidence.

A

The correct answer is: Overrule it, because the girlfriend’s testimony is admissible habit evidence.

Discussion of correct answer: Generally, character evidence is not admissible to prove action in conformity with that character. However, an exception to this rule exists for habit evidence, which refers to actions that a person “always” or “invariably” performs. Actions performed “frequently” or “often” are not habits; a habit is a regular response to a given situation that is performed without must forethought. Here, the girlfriend’s testimony is that she “always” took the necklace off before bed and put it in the same location. The girlfriend’s testimony is admissible as habit evidence.

How well did you know this?
1
Not at all
2
3
4
5
Perfectly
56
Q

A man pulled his truck onto the freeway and collided with a car. The driver of the car was seriously injured in the accident, and subsequently filed suit against the truck driver, alleging that he failed to yield as he entered the freeway from the entrance ramp. The truck driver, in turn, claimed that the driver of the car was speeding and had failed to make any attempt to avoid the accident even though the two lanes to the left of the entrance ramp were wide open. The truck driver’s attorney called the truck driver to testify that when he approached the car immediately following the accident, the driver of the car stated over and over, “I should have slowed down or moved over.” The driver of the car objected.

Should the truck driver be allowed to testify as to the driver of the car’s statement?

A No, because it is inadmissible hearsay.
B No, because the truck driver is an interested party and there is no corroboration of the statement.
C Yes, because it shows awareness by the driver of the car that he was speeding.
D Yes, because it is a statement by an opposing party.

A

The correct answer is: Yes, because it is a statement by an opposing party.

Discussion of correct answer: Under Federal Rule of Evidence 801, a statement by an opposing party that is offered against that party is nonhearsay. Here, the car driver’s statement constitutes an opposing party’s statement, and will therefore be admissible against him at trial.

How well did you know this?
1
Not at all
2
3
4
5
Perfectly
57
Q

A murder mystery writer is charged with the murder of her husband. The prosecution charged that the writer poisoned her husband by mixing yew berries into his Thanksgiving cranberry relish. This was the same poison used in the writer’s recent murder mystery novel. The prosecution also alleges the writer wanted to marry her editor but feared a divorce from her husband might affect her popularity and the sales of her books.
The defense called the husband’s personal physician to the stand as its first witness. During examination, the writer’s lawyer asked, “Isn’t it true that the husband’s cause of death is unknown?” The doctor responded, “No, the husband died after ingesting poison.” The writer’s lawyer then attacked the testimony as a recent fabrication and asked, “Isn’t it true that you stated on the husband’s death certificate that the cause of death was unknown?” The prosecutor objected. The judge’s ruling will be that the last question is

a. proper because the doctor’s testimony about written opinions is more reliable.
b. proper, because a party may impeach its own witness.
c. not proper, because a party may not impeach its own witness with a prior inconsistent statement.
d. not proper, because the doctor is a general practitioner rather than a toxicology expert.

A

b. proper, because a party may impeach its own witness.

How well did you know this?
1
Not at all
2
3
4
5
Perfectly
58
Q

One night, a man and his friend were shooting pool and drinking beer when the man received a call on his cell phone from his wife. After finishing his conversation with his wife, the man told his friend that he had to wrap up the game because his wife had just gotten into a fistfight with a neighbor, and it appeared that his wife was the instigator. When the man arrived home, he spoke with a bystander and discovered that his wife hadn’t been the instigator after all. The neighbor sued the wife for personal injuries. At trial, the husband took the stand, but denied ever having told his friend that his wife was the instigator of the fight. The neighbor now seeks to call the friend to testify as to the husband’s comments at the pool hall. Should the court allow the friend’s testimony?

a. No, as hearsay not within an exception.
b. No, because the testimony calls for an opinion.
c. Yes, to impeach the husband.
d. Yes, as a statement.

A

c. Yes, to impeach the husband.

How well did you know this?
1
Not at all
2
3
4
5
Perfectly
59
Q

A girlfriend and boyfriend lived together and had a tumultuous relationship. One night, the couple got in a fight. The next morning, the girlfriend realized that the boyfriend had moved out. The girlfriend also noticed that her valuable diamond necklace that she had inherited from her grandmother was gone. The girlfriend sued the boyfriend for conversion of the diamond necklace. At trial, to prove the location of the necklace before it went missing, the girlfriend testified that she wore the necklace everyday, and she always took the necklace off before she went to bed and placed it in the same place in her jewelry box. No other witness can attest to the girlfriend’s testimony. The boyfriend’s attorney objected to the girlfriend’s testimony regarding the necklace. How should the judge rule on the objection?

a. Overrule it, because the girlfriend’s testimony is relevant to whether the boyfriend converted the necklace.
b. Overrule it, because the girlfriend’s testimony is admissible habit evidence.
c. Sustain it, because the girlfriend’s testimony cannot be corroborated.
d. Sustain it, because the girlfriend’s testimony is inadmissible character evidence.

A

b. Overrule it, because the girlfriend’s testimony is admissible habit evidence.

How well did you know this?
1
Not at all
2
3
4
5
Perfectly
60
Q

The plaintiff owned a very successful seafood restaurant. At the end of the previous season, the plaintiff had become concerned about competition from a new restaurant approximately two miles away. Now, at the beginning of this new season, the plaintiff was particularly concerned, because the clams and scallops he was receiving from his supplier were not up to their usual standard. The plaintiff believed that the diminished quality was due to changes in water temperatures and environmental conditions, which the plaintiff expected to improve by the start of summer. However, even well into the summer season, the plaintiff found that the seafood continued to be subpar.

Early in July, he hired a secretary, who was previously employed by the supplier, to work in his office. Shortly after the secretary began working for the plaintiff, she told him that the owner of the new restaurant was an old high school buddy of the president and owner of the supplier. The secretary told the plaintiff that the owner of the supplier was a silent partner in the new restaurant, and that at a meeting in March, the secretary overheard them discussing a plan to supply inferior seafood to the plaintiff’s restaurant, in order to hurt the plaintiff’s reputation and increase business at the new restaurant.

The plaintiff immediately instructed his staff to keep tallies of how many meals containing seafood supplied by the supplier were sent back to the kitchen as a result of customer complaints. In a subsequent action by the plaintiff against the supplier for interference with business relations, the plaintiff moved to introduce the tallies into evidence.

Are the tallies admissible?

A Yes, because they constitute a past recollection recorded.
B Yes, because they are a business record.
C No, because they are hearsay not within any exception.
D No, because there is no proof that the meals were sent back because of the inferior seafood.

A

The correct answer is:No, because they are hearsay not within any exception.

Discussion of correct answer:Hearsay is an out-of-court statement offered into evidence to prove the truth of the matter asserted. Here, the tallies are clearly written testimony being presented for the purpose of proving that the supplier was supplying the plaintiff’s restaurant with inferior seafood. Therefore, the tallies are inadmissible hearsay if they are not subject to a recognized hearsay exception. Moreover, because the tallies were clearly created for the purpose of the plaintiff’s litigation and not in the ordinary course of the plaintiff’s business, they do not come within the business records exception to the hearsay rule.

How well did you know this?
1
Not at all
2
3
4
5
Perfectly
61
Q

A chef was involved in a two-car collision and sued the other driver, a cook. At the trial, the chef testified that the cook failed to stop at a stop sign and hit her, so the accident was not her fault. The cook called a witness who testified that she has known the cook for many years, and she observed that he always completely stopped at every stop sign. Is the witness’s testimony admissible?

a. Yes, because it tends to establish a pertinent habit of the cook’s while driving.
b. Yes, because it is evidence of the cook’s cautious character behind the wheel.
c. No, because the cook’s reputation for careful driving can only be proved by opinion or reputation.
d. No, because the cook’s prior conduct cannot be used to prove he is a careful driver.

A

a. Yes, because it tends to establish a pertinent habit of the cook’s while driving.

How well did you know this?
1
Not at all
2
3
4
5
Perfectly
62
Q

A patient sued his doctor for malpractice. The patient called an expert witness to testify that the drugs prescribed by the doctor were so experimental that it constituted negligence under accepted practices in the medical community. On cross-examination, the expert stated that Pharmacology: A Guide for the Practitioner was a reliable authority in her area of specialty. The expert testified that she, however, did not rely on that treatise in forming her opinion. The doctor’s attorney then proposed to read a portion from the treatise, which stated that the drugs prescribed by the doctor are widely used by other physicians in treating patients. The patient’s attorney objected.

Will the court admit the evidence?

A No, because the text of the treatise constitutes inadmissible hearsay.
B No, because the expert did not rely on the treatise in forming her opinion.
C Yes, as substantive evidence.
D Yes, to impeach the expert, but not as substantive evidence.

A

The correct answer is: Yes, as substantive evidence.

Discussion of correct answer: FRE 803(18) provides that statements contained in treatises may be admitted into evidence during direct or cross-examination of an expert witness if: (1) the treatise is established as a reliable authority; and (2) the treatise is called to the attention of the expert witness during cross-examination or is relied upon by the expert in direct testimony. As an exception to the hearsay rule, statements in treatises admitted pursuant to FRE 803(18) may be used as substantive evidence. They may be read to the jury.

How well did you know this?
1
Not at all
2
3
4
5
Perfectly
63
Q

A woman was on trial for the murder of her husband. The prosecution alleged that the woman paid one of her neighbors $25,000 to kill her husband. Immediately after the killing, the neighbor boarded a bus and called his roommate. In a lowered voice, he whispered, “Pack your bags, we gotta get out of town for a while ‘till the heat cools off. Don’t worry about money, I got it covered.” The neighbor’s conversation was heard by a fellow passenger. The neighbor was arrested at the airport the following day. At trial, the prosecution attempted to call the neighbor to testify, but he invoked his Fifth Amendment right not to testify. The prosecution then called the passenger to testify. The wife objected.

Which of the following is most accurate?

A The statement is inadmissible, because it does not directly incriminate the neighbor.
B The statement is inadmissible, because it does not directly incriminate the wife.
C The statement is admissible, as a statement against interest.
D The statement is admissible, as an excited utterance.

A

The correct answer is: The statement is admissible, as a statement against interest.

Discussion of correct answer: If a declarant makes a statement against his penal interest and is unavailable for trial, the statement will qualify as a statement against interest and will be admitted as an exception to the hearsay rule. The facts indicate that the neighbor has invoked his Fifth Amendment right not to testify. The neighbor’s statement also meets this exception’s criteria that it was against the neighbor’s penal interest when he said it, and that a reasonable person would not have made the statement if it were not true. Therefore, this is the best answer.

How well did you know this?
1
Not at all
2
3
4
5
Perfectly
64
Q

One morning, a man and a woman were seated at the kitchen table, eating breakfast together. The man said to the woman, “I had the funniest sensation last night…a burning feeling right in the middle of my forehead. It was where the doctor attached that probe to my head during my check-up yesterday. It lasted about 10 minutes, and then went away. It was weird.” She nodded her head, took another bite of her toast and replied, “That’s nice, dear.” A few minutes later, the man collapsed and died. The woman sued the doctor for medical malpractice. At trial, her attorney asks her to repeat what her husband said to her at breakfast.

How should the judge rule on the admissibility of this statement?

A Admit the statement, because it qualifies as a dying declaration.
B Admit the statement, because it qualifies as a statement of then-existing physical condition.
C Exclude the statement, because the man was not under the stress of excitement caused by a startling event.
D Exclude the statement, because it is hearsay not within any exception.

A

The correct answer is: Exclude the statement, because it is hearsay not within any exception.

Discussion of correct answer: Hearsay is a statement made outside of court that is offered into evidence in order to prove the truth of the matter asserted. The husband’s statement to the wife is hearsay if it is offered to prove that he suffered pain at the spot where the doctor attached the probe. It is inadmissible because it does not fit any exceptions to the hearsay rule. It is not a dying declaration, because there is no evidence the husband believed he was dying at the time he made the statement. It is not a statement of then-existing physical condition, because the husband is describing a past condition and not a present condition. It is not an excited utterance, because there is no evidence that the husband was under the stress of excitement caused by a startling event.

How well did you know this?
1
Not at all
2
3
4
5
Perfectly
65
Q

A father gives his son the keys to the father’s sports car. Without the father’s knowledge or permission, the son takes it street racing and in running a red light, he injures the innocent driver of another car. The innocent driver sues the father on a negligent entrustment theory. At trial, the innocent driver calls a neighbor of the father and son who testifies only that the son had always been “an extremely reckless sort of kid” whom the entire neighborhood regarded as having “absolutely no regard for the safety of others.” The defense objects. Is the neighbor’s testimony proper?

a. No, the neighbor did not support her testimony with specific instances of reckless conduct.
b. No, a witness may not testify as to the ultimate issue.
c. Yes, the son’s character is an essential element of the innocent driver’s cause of action.
d. Yes, the neighbor’s testimony did not offer evidence of specific instances of reckless conduct.

A

c. Yes, the son’s character is an essential element of the innocent driver’s cause of action.

How well did you know this?
1
Not at all
2
3
4
5
Perfectly
66
Q

The defendant is on trial for bank robbery. The prosecutor calls a teller as a witness. The teller testified that a man entered the bank on the day in question, pointed a gun at her and demanded money. She was asked if she recognized the person who robbed her among the people sitting in the courtroom. She pointed to the defendant. The defense attorney did not cross-examine the teller. During the defendant’s case-in-chief, his attorney called the teller’s optometrist to the stand. The optometrist testified that the teller was nearsighted and even with glasses, her vision was 20/60 in both eyes. The prosecutor objected to this testimony. Is the testimony admissible?

a. Yes, as extrinsic impeachment evidence.
b. Yes, as an exception to the hearsay rule.
c. No, as hearsay.
d. No, as improper impeachment evidence.

A

a. Yes, as extrinsic impeachment evidence.

67
Q

A man was prosecuted for assault and battery after he admitted to striking his brother with a baseball bat. The man claimed that he acted in self-defense after he was attacked by his brother, who was calling him a cheater and coming after him with a golf club. As his first defense witness, the man called a neighbor to testify that the man has always been a good neighbor. Is the witness’s testimony admissible?

a. No, because it is merely the opinion of a character witness.
b. No, because it is not directed toward a pertinent trait of the man’s character.
c. Yes, because it is relevant to show the man’s character.
d. Yes, because it is relevant to support the man’s credibility.

A

b. No, because it is not directed toward a pertinent trait of the man’s character.

68
Q

While the defendant was awaiting trial on murder charges, another inmate at the jail asked to speak to a police investigator. The inmate told the investigator that he was suffering pangs of conscience because he had actually committed the murder, not the defendant. The investigator did not believe the inmate, but duly recorded what the inmate had said. Prosecutors provided this information to the defendant’s attorney. In the meantime, before trial, the inmate died of natural causes in his sleep. At the defendant’s murder trial, the defendant called the investigator to the stand to tell the jury what the inmate had told him. The defendant offered this into evidence as a statement against interest under Rule 804. There was no objection by the prosecution. During the prosecution’s rebuttal case, the prosecutor calls one of the inmate’s former cellmates to testify that the inmate had a reputation for being untruthful. The defense objects.

How should the judge rule on the defense objection?

A Sustained, because it is improper to introduce reputation or opinion testimony to attack the character of a witness who has not personally testified in the courtroom.
B Sustained, because the inmate is dead and cannot rebut the attack on his character.
C Overruled, because the prosecution did not object to the defendant introducing the inmate’s statement.
D Overruled, because the evidence is proper character evidence to attack the credibility of a hearsay declarant.

A

The correct answer is: Overruled, because the evidence is proper character evidence to attack the credibility of a hearsay declarant.

Discussion of correct answer: Rule 806 permits attacks on the credibility of a hearsay declarant in the same manner as any other witness’s character could be attacked. The character for truthfulness of all witnesses is relevant at trial and admissible under Rule 608 and may be introduced in the form of reputation or opinion testimony. Thus, the prosecution’s use of a former cellmate to testify about the inmate’s reputation for untruthfulness is proper, even though the inmate did not personally testify at trial.

69
Q

A graduate student severely cut her arm on a rough metal edge on the door of a carnival roller coaster. She filed a negligence action against the carnival owner. The student’s lawyer calls a witness to testify that she was cut by the same rough metal edge two months before the student was injured. The witness further testified that she notified the carnival owner about the problem with the rough metal edge before the student was injured. The carnival owner’s lawyer learns on cross-examination of the witness that the witness and the student used to be roommates. The student’s lawyer then seeks to establish on redirect that the student and the witness have not even seen each other for five years. Should the court admit the witness’s testimony on this point?

a. Yes, to rebut an inference of bias.
b. Yes, to establish the witness’s state of mind.
c. No, because it is not relevant to the issue of the rough metal edge.
d. No, because it is hearsay not within a recognized exception.

A

a. Yes, to rebut an inference of bias.

70
Q

A musical instrument company is sued when one of its delivery trucks is involved in an accident with a passenger car. At trial, the plaintiff asserts that the company failed to properly maintain the brakes on the delivery truck. The company calls its head mechanic to testify that all of the company’s delivery trucks undergo a safety inspection, including an inspection of the brakes, every morning before the trucks leave the yard. He supervises the inspectors, but does not perform the inspections himself, and was not present in the yard on the morning of the accident.

Is the head mechanic’s testimony admissible?

A Yes, because it is evidence of the routine practice of a business.
B Yes, because it is evidence of the company’s propensity for safe conduct.
C No, because the head mechanic does not have personal knowledge that the vehicle was inspected that morning.
D No, because only personal habits are admissible to show that a defendant acted according to habit on a particular occasion.

A

The correct answer is: Yes, because it is evidence of the routine practice of a business.

Discussion of correct answer: Under FRE 406, evidence of the habit of a person or the routine practice of an organization is admissible to prove that the conduct or routine on a particular occasion was in conformity, whether corroborated or not and regardless of the presence of an eyewitness on a particular occasion conformed to that habit. Thus, this answer is correct. The head mechanic may testify as to the company’s daily inspections because the inspections are a routine practice of the business.

71
Q

A group of women having a bachelorette party were inside of a club when a shooting occurred behind their table. The bartender was arrested for aggravated assault for the shooting. At trial, the bride-to-be took the stand to testify. She admitted that she did not actually see the shooting but that she heard someone behind her yell, “Watch out! He’s got a gun! The bartender’s got a gun!” She testified that she heard this being yelled at the same time that she heard the gunshots. There was only one bartender, a male, employed by the bar that night. The person yelling never came forward to speak to the police about the shooting.

Can the bride-to-be testify about what she heard?

A No, because the declarant’s statement it is hearsay not within any exception.
B No, because the declarant is unavailable.
C Yes, because the statement describes an event while the declarant was perceiving that event.
D Yes, because there is sufficient corroboration presented, therefore this is a question of weight, not admissibility.

A

The correct answer is: Yes, because the statement describes an event while the declarant was perceiving that event.

Discussion of correct answer: This answer is the most accurate because it presents a portion of the definition of the present sense impression exception to the hearsay rule. With this exception, a statement which describes or explains an event or condition made while the declarant was perceiving the event or condition, or immediately thereafter, are not excluded by the hearsay rule whether the declarant is available to testify or not. Here, the unidentified declarant yelled out statements regarding the shooter in a club. The statements essentially identified the shooter. The statements were made contemporaneous to the shooting and while the declarant perceived the shooting. Therefore, the Present Sense Impression exception would apply and allow the bride-to-be to testify about the statements in order to prove the truth of the matter asserted.

72
Q

A defendant was on trial for murdering his girlfriend. She was found shot to death in her bedroom. While the first officer to arrive was in the bedroom he saw the girlfriend’s open diary on her desk. The diary, which was confiscated by the police, was properly authenticated as being in the girlfriend’s handwriting. During the defendant’s murder trial, the prosecution called the officer to testify. He proposed to testify that when he looked at the open diary, he saw the following handwritten passage: “My boyfriend threatened to kill me today.”

Upon objection, is the officer’s proposed testimony admissible?

a. Yes, because the diary was in plain view.
b. Yes, because he has firsthand knowledge of the diary.
c. No, because the best evidence rule requires the diary to be produced into evidence.
d. No, because the passage in the diary is hearsay not within any recognized exception.

A

d. No, because the passage in the diary is hearsay not within any recognized exception.

73
Q

A father gives his son the keys to the father’s sports car. Without the father’s knowledge or permission, the son takes it street racing and in running a red light, he injures the innocent driver of another car. The innocent driver sues the father on a negligent entrustment theory. At trial, the innocent driver calls a neighbor of the father and son who testifies only that the son had always been “an extremely reckless sort of kid” whom the entire neighborhood regarded as having “absolutely no regard for the safety of others.” The defense objects.

Is the neighbor’s testimony proper?

A No, the neighbor did not support her testimony with specific instances of reckless conduct.
B No, a witness may not testify as to the ultimate issue.
C Yes, the son’s character is an essential element of the innocent driver’s cause of action.
D Yes, the neighbor’s testimony did not offer evidence of specific instances of reckless conduct.

A

The correct answer is: Yes, the son’s character is an essential element of the innocent driver’s cause of action.

Discussion of correct answer: As a general rule, evidence of a person’s character may not be offered to prove that the person committed a particular act. The testimony here, however, is not excluded by this rule because it is not being offered to prove that the son committed any particular act (i.e., that the son drove recklessly or that the son in fact injured the innocent driver). Instead, the testimony is offered to prove what the father should have known about his son. The testimony is directly relevant because this is a negligent entrustment claim and so the plaintiff must show that the father knew or should have known that the son had a propensity to drive recklessly. Because the son’s propensity for recklessness is directly at issue, the testimony is proper.

74
Q

A man crashed into a woman’s car. Shortly after the accident, a police officer arrived at the scene, and the woman told the officer that the man had run the red light and swerved into her car. The police officer wrote her statement in the “Accident Report Form,” which he was required to complete after every accident. The woman sued for personal injuries sustained in the accident. On direct examination, her attorney calls her to testify as to her statement to the police officer.

Is the woman’s statement admissible?

A No, because a party is precluded from introducing their out-of-court statements at a subsequent trial.
B No, because it is hearsay not within any exception.
C Yes, because it constitutes a prior consistent statement.
D Yes, because the woman is in court testifying to an out-of-court statement, and is therefore subject to cross-examination.

A

The correct answer is:No, because it is hearsay not within any exception.

Discussion of correct answer:The woman’s statement was made out of court and is now being offered for the truth of the matter asserted therein (i.e., that the man had run a red light and swerved into her car). The statement is therefore hearsay not within any applicable exception. Thus, the statement is inadmissible.

75
Q

A chemist is on trial for assaulting his research assistant. Both the chemist and the research assistant work for a large university. A week after the assault occurred, the research assistant contacted a department manager and told the manager that the chemist had lost his temper and thrown a crate of test tubes containing hazardous materials at her. The manager jotted down notes during the meeting about what the research assistant said. At the chemist’s trial, the prosecution wishes to offer the handwritten notes during the manager’s testimony. The defense objects on the grounds that the statement is inadmissible hearsay.

Will the objection be sustained?

A Yes, because the statement is hearsay without an exception.
B Yes, because the research assistant is the proper party to testify about her own statements.
C No, because the statement was made by a party-opponent.
D No, because the manager is giving live testimony and the notes fall under the business records exception to the hearsay rule.

A

The correct answer is: Yes, because the statement is hearsay without an exception.

Discussion of correct answer: The manager’s notes are hearsay within hearsay which is prohibited under FRE 805 unless all levels of the hearsay fall within an exception. The first level of hearsay is the statement by the research assistant about what happened. She made an out of court statement that the chemist threw test tubes at her, and if it is being offered to prove that the chemist assaulted her, it is hearsay. The second level of hearsay is the manager’s notes. His notes are a statement in and of themselves. Even if the manager’s notes qualified for a hearsay exception like the business records exception, the original statement by the research assistant is inadmissible and the notes containing the statement are therefore also inadmissible. Thus, this answer is incorrect because the notes are hearsay within hearsay without an exception.

76
Q

The defendant is on trial for murder. During his case-in-chief, he offers the following evidence from a friend: “I have known the defendant all of my life, and he has a reputation in the community as a peaceful man.” Without cross-examining the friend, the prosecutor then offers evidence during her rebuttal case. She calls a witness, who is an acquaintance of the friend, to testifies as follows: “I have known the friend for 20 years, and on at least 10 occasions I have heard the friend lie to other people about important matters.” The defense attorney objects. Is the testimony admissible?

a. No, as improper rebuttal of character evidence.
b. No, as improper impeachment.
c. Yes, as character evidence.
d. Yes, for impeachment only.

A

b. No, as improper impeachment.

77
Q

An actress was planning her wedding to a businessman. The wedding planning and the wedding were to air as a television series. The actress’s neighbor wrote on the neighbor’s blog that the actress and the businessman had already gotten married at the courthouse three years ago, and they were only doing the television series for money and attention. The actress sued the neighbor for defamation. The neighbor contended that the statements were true.

At trial, is the marriage certificate admissible to prove the date that the actress and the businessman were married?

A No, because the document is fewer than 20 years old.
B No, because the marriage certificate was not authorized by a religious organization.
C Yes, because the marriage was performed at the courthouse.
D Yes, because character is an ultimate issue is a defamation case.

A

The correct answer is: Yes, because the marriage was performed at the courthouse.

Discussion of correct answer: Hearsay is an out-of-court statement offered to prove the truth of the matter asserted. Hearsay includes written documents that are offered to prove the truth of the matter asserted in the writing. Hearsay is inadmissible unless an exemption or an exception applies. A hearsay exception exists for a statement of fact contained in a certificate of marriage that was made by a person who was authorized by law to perform the marriage. A marriage certified at a courthouse is authorized by law, and this hearsay exception applies to admit the marriage certificate to prove the date that the actress and the businessman were married.

78
Q

At the trial of a woman’s personal injury action against a driver, a pedestrian who was near the accident scene, but was checking her e-mail on her cell phone at the time and did not see what happened, testifies that a bystander who witnessed the accident shouted immediately after the accident occurred: “Good Lord! The blue coupe just ran through a red light and hit the red pickup!” Previous evidence had established that the driver drove a blue coupe and the woman a red pickup. The driver offers to call an investigator who will testify that he spoke with the bystander the next day, and the bystander said that the light was green when the blue coupe drove through the intersection. The bystander died prior to trial. Should the investigator’s testimony be admitted over objection?

a. No, because the bystander is not available to explain or deny the contradiction.
b. No, because it is hearsay not within any exception.
c. Yes, for the purpose of impeachment and as substantive evidence.
d. Yes, for the purpose of impeachment only.

A

d. Yes, for the purpose of impeachment only.

79
Q

A rancher and his wife live on 20 acres of land that they use to raise cattle. Around the holidays, they also use the ranch as a petting zoo. During a trip to the rancher’s petting zoo, a child was severely burned when a gas line exploded. The rancher and his wife were sued by the family of the injured child. The rancher died of a heart attack the week before trial. As evidence that she was not liable for the child’s injuries, the rancher’s wife presented several letters that the rancher wrote to his son stating that he knew the true owner of the ranch had moved out of state and that the rancher and his wife were squatting on the property in-between the true owner’s visits to the ranch to try and make money off of the cattle and petting zoo.

Are the letters admissible to prove that the rancher and his wife did not own the property?

A Yes, because the rancher died before trial.
B Yes, because the wife is not the declarant.
C No, because the letters cannot be authenticated.
D No, because the rancher is unavailable for trial.

A

The correct answer is: Yes, because the rancher died before trial.

Discussion of correct answer: This question tests the reader’s knowledge of the various requirements for the exceptions to the hearsay rule under FRE 803 and FRE 804. All of the exceptions under FRE 804 require that the declarant be unavailable. One important exception is for statements made against interest. Basically, if a declarant has made a statement that is so contrary to his interests that the truth of the statement is not in question, the statement is admissible hearsay. Again, the exception only applies if the declarant is unavailable. Here, the rancher tells his son in the letters that the rancher and his wife are committing several violations of the law and subjecting themselves to civil liability if they were to get caught. Not only could the rancher and his wife be removed from the property but they could face civil damages and criminal charges for their actions. These are the types of consequences considered by the statement against interest exception and since the rancher is unavailable, his letters are admissible hearsay.

80
Q

In a suit by an investor against a speculator, a relevant fact is the price of gold as quoted on a specific date 20 years ago. The investor calls a librarian to the stand to authenticate a microfilm copy of a newspaper from that date. This copy, kept in the archives of the public library, is the only record of that particular issue of the newspaper that either party has been able to locate.

Is the microfilm copy admissible?

a. Yes, because there is no indication that the original is unavailable.
b. Yes, as an ancient document.
c. No, as hearsay not within any exception.
d. No, because it violates the best evidence rule.

A

b. Yes, as an ancient document.

81
Q

In an employee’s wrongful discharge action against her employer, the employer alleges that the employee falsified her employment history before joining the employer’s staff. Her resume, admitted into evidence without objection, shows that she worked for the federal Department of the Interior as a purchasing agent for three years. The employer sought to introduce payroll records from the Department of the Interior for that entire period, showing there had never been a paycheck issued to someone with the employee’s name.

Is this evidence admissible?

A No, because the prejudice outweighs the probative value of the records because she may have had another name during that time period.
B No, unless the person who prepared the records will testify.
C Yes, although they can only be admitted in summary form to prevent wasting the court’s time.
D Yes, provided an appropriate person lays a foundation for the records.

A

The correct answer is: Yes, provided an appropriate person lays a foundation for the records.

Discussion of correct answer: Federal Rule of Evidence Rule 803(7) allows the use of records to prove absence of an entry in the records as long as the documents satisfy the requirements for records of a “regularly conducted activity”: there must be a regular business practice to make such records; they must be made at or near the time of the event and be made by a person with knowledge of the matters therein. This foundation may be established by the testimony of the custodian of the records or other qualified person.

82
Q

A state law provides that a non-deadly assault on a public employee while the employee is performing his duties is a misdemeanor. A social activist and demonstrator was expressing his views on toxic waste when a city trash collector, who was collecting trash at the time, approached the activist in an attempt to engage him in a debate about the finer points of waste management. The debate soon escalated into a fistfight. The activist was arrested and charged with violation of the aforementioned statute.

Pursuant to a court order, the city sanitation department produced the trash collector’s confidential personnel file, which contained three reports of prior incidents of aggressive behavior upon citizens while he was collecting trash. The activist was ultimately acquitted of misdemeanor battery, and shortly thereafter, brought a civil action against the trash collector for damages. During his case-in-chief, the activist offers the testimony of a witness who plans to testify that she observed the trash collector hit another demonstrator for no reason, about one month prior to this incident. That demonstrator was hospitalized with serious injuries. The trash collector’s counsel objects. Should the trial court sustain the objection?

a. Yes, because the witness’s testimony is irrelevant.
b. Yes, because the witness’s testimony constitutes inadmissible character evidence.
c. No, because the witness’s testimony is relevant to a material issue in the case.
d. No, but it should limit the witness’s testimony to the issues of damages only.

A

b. Yes, because the witness’s testimony constitutes inadmissible character evidence.

83
Q

Which of the following is most likely to be admissible under the dying declaration exception to the hearsay rule?

a. Statement by a husband who, after his car accident and moments before his death, said, “I wish I remembered to say goodbye to my wife.”
b. Statement by a defendant who, wrongly believing death was imminent, said, “My father shot me.”
c. Statement by a victim who, before falling into an irreversible coma caused by a bullet wound, said, “My wife shot me.”
d. Statement by unavailable declarant made six weeks after a near-death accident that his brother attempted to kill him.

A

c. Statement by a victim who, before falling into an irreversible coma caused by a bullet wound, said, “My wife shot me.”

84
Q

A defendant was prosecuted for assault and battery after she admitted striking her sister with a broomstick that was part of her Halloween witch’s costume. The defendant claimed she acted in self-defense after she was attacked by her sister, who was calling her a “witch” and coming after her with a kitchen knife. As her first defense witness, the defendant calls a witness to testify that the defendant is a good solid neighbor. Is the witness’s testimony admissible?

a. Yes, because it is relevant to show the defendant’s character in this case.
b. Yes, because it is relevant to support the defendant’s credibility.
c. No, because it is merely the opinion of a character witness.
d. No, because it is not directed toward a pertinent trait of the defendant’s character.

A

d. No, because it is not directed toward a pertinent trait of the defendant’s character.

85
Q

A plaintiff filed a negligence action against a defendant arising out of an automobile accident. During trial, the plaintiff called the defendant as an adverse witness. During direct examination, plaintiff’s counsel asked the defendant whether he drove through the red light and struck the plaintiff’s car. The defendant denied doing so and claimed that the light was green when he entered the intersection. The plaintiff then called a police officer to testify that when he spoke to the defendant immediately after the accident, he said that he was sorry for driving through the red light. The defendant’s counsel objected to the testimony of the police officer.

Is the testimony admissible?

A Yes, as a prior inconsistent statement.
B Yes, as both substantive and impeachment evidence.
C No, because it is improper extrinsic evidence.
D No, as hearsay not within an exception.

A

The correct answer is: Yes, as both substantive and impeachment evidence.

Discussion of correct answer: A statement of a party that is offered in evidence by the opponent party is not hearsay and freely admissible. They are admissible substantively, that is, they may be used by the jury to decide the lawsuit. Unlike impeachment evidence, which is admissible for the more limited purpose of challenging the credibility of a witness, substantive evidence may be used by the jury to determine liability and damages in a civil case, and guilt or not in a criminal case. Under the facts of this question, the statement may also be used to undermine the credibility of the defendant.

86
Q

The defendant is charged with shooting and killing a victim with the defendant’s .357 Magnum revolver. According to evidence obtained by police investigating the case, the victim showed up drunk at the defendant’s apartment. When the defendant opened the front door, the victim began yelling at the defendant. The defendant was unable to exit the apartment, because the victim was blocking the door, and there was no alternate exit route. When the victim began yelling even more loudly and threateningly, the defendant drew his gun and shot at the victim from the back corner of the living room. The bullet struck the victim in the jugular vein. The defendant called for medical assistance, but the victim bled to death before the ambulance arrived. The defendant was charged with murder and has asserted self-defense. Upon commencement of his trial, the defendant immediately attempts to call to the stand various witnesses to testify to the victim’s reputation as a belligerent and intimidating man who regularly terrorized the neighborhood. The defendant also presents testimony including specific instances in which the victim terrorized multiple neighbors, including the defendant. Prior to the witnesses’ testimony, the state moves to bar admission of all the testimony concerning the victim’s character and reputation.

How should the judge rule?

A The evidence is admissible, because it is relevant to the defendant’s state of mind, and, therefore, to his self-defense claim.
B The evidence is inadmissible, because character evidence cannot be used to show that a party acted in conformance therewith on a specific occasion.
C The evidence is inadmissible, unless the testifying witnesses were present for all the events.
D The evidence is inadmissible, because the victim is not available to defend his reputation.

A

The correct answer is:The evidence is admissible, because it is relevant to the defendant’s state of mind, and, therefore, to his self-defense claim.

Discussion of correct answer:While the introduction of character evidence to prove that a party acted in conformance therewith on a particular occasion is generally prohibited under Federal Rule of Evidence 404, such evidence may be admissible under certain particular circumstances, as where an accused seeks to introduce pertinent evidence of the character of the victim in support of a claim of self-defense to a charge of homicide. Here, the defendant was charged with the victim’s homicide, and evidence of the victim’s reputation for violence is relevant to the validity of the defendant’s state of mind and his claim of self-defense. As such, the evidence is admissible.

87
Q

A defendant was charged with the armed robbery of an elderly woman. The defendant denied all the allegations and pleaded not guilty. At trial, the defendant called his friend as a witness. The friend was to testify that the defendant’s had a reputation as “a gentle person who abhors violence.” Should the court allow the friend’s testimony to be admitted as evidence at trial?

a. No, because the defendant has not yet testified.
b. No, because the friend’s bias makes him unreliable.
c. Yes, because it tends to prove that the defendant is not the type of man who would commit armed robbery.
d. Yes, because it is relevant to the question of whether the defendant’s denial of guilt can be believed.

A

c. Yes, because it tends to prove that the defendant is not the type of man who would commit armed robbery.

88
Q

A college student in Illinois walked into his parents’ room one evening and said, “Tomorrow my roommate and I are flying to Las Vegas. We’re going to gamble and take in a show.” His parents looked up, smiled, and told him to have a good time and to be careful. It was the last time they ever saw their son alive. Their son’s dismembered body was found in the desert outside of Las Vegas. Shortly after the son’s body was discovered, police in Illinois arrested the student’s roommate. He was extradited to Las Vegas and charged with the murder of the college student. At trial, the prosecutor called the parents to testify about what their son had told them the evening he came to their room, as proof of the date on which their son had traveled to Las Vegas.

Should this statement be admitted at trial to prove that the college student travelled to Las Vegas on the date he stated?

A Yes, because a statement of intent to engage in conduct is admissible to prove the subsequent conduct of the declarant.
B Yes, because the statement is proof of a material fact and is more probative on the point for which it is offered than any other evidence the prosecution could procure through reasonable efforts.
C No, because the student’s roommate is not unavailable to testify as to the date on which the student travelled to Las Vegas.
D No, because the statement is hearsay not within any exception.

A

The correct answer is: Yes, because a statement of intent to engage in conduct is admissible to prove the subsequent conduct of the declarant.

Discussion of correct answer: Rule 803, which lays out certain exceptions to the hearsay rule, provides for the admissibility of statements of intent as proof of the declarant’s then-existing mental condition. These statements of intent can be offered to prove the declarant’s subsequent conduct. Here, the college student made a statement of intent to his parents. Therefore, the statement may be used to prove that he travelled to Las Vegas on the day he said he would.

89
Q

A defendant was charged with molesting his girlfriend’s 10-year-old son. At trial, the prosecution calls a witness to testify that 11 years ago, when the witness was 14 years old, the defendant was dating the witness’s mother, and the defendant molested the witness. The witness did not report the molestation when it occurred.

Is the witness’s testimony admissible?

A No, because the witness’s testimony is inadmissible character evidence.
B No, because the alleged molestation of the witness occurred 11 years ago.
C Yes, because the probative value of the witness’s testimony substantially outweighs the danger of unfair prejudice to the defendant.
D Yes, because the witness’s testimony tends to prove the defendant’s propensity to molest children.

A

The correct answer is: Yes, because the witness’s testimony tends to prove the defendant’s propensity to molest children.

Discussion of correct answer: Character evidence is generally inadmissible to prove that a person had a propensity to act a certain way. However, the Federal Rules of Evidence have special rules for cases involving sex offenses. Where a defendant is accused of child molestation in a criminal case, evidence that the defendant committed any other child molestation is admissible and may be considered on any matter to which it is relevant. Evidence that the defendant molested another child is relevant to prove that the defendant had a propensity to molest children. Such evidence is subject to the 403 balancing test, but it will not be excluded merely because it introduces propensity evidence.

90
Q

A mother brought a personal injury action on behalf of her son against a cab company for damages arising from a car accident in which the mother’s car collided with a cab driven by one of the company’s drivers. During his lunch break, the cab driver had consumed several alcoholic drinks.
In her complaint, the mother alleged that the cab company had been negligent in approving the hiring of the cab driver, who was an alcoholic, and that the company reasonably should have known that the driver’s alcoholism made him a serious risk to others while operating a motor vehicle. At trial, the mother called the cab driver’s roommate to testify that she had lived with the driver for the six months prior to the accident and that, on three separate occasions, she had observed the driver operate a motor vehicle while in a severely intoxicated state. The cab company objected to the introduction of the roommate’s testimony. Should the court admit the roommate’s testimony over the cab company’s objection?

a. No, because evidence of specific instances of misconduct may only be inquired into on cross-examination and may not be proven by extrinsic evidence.
b. No, because the cab driver has not testified that he has never operated a motor vehicle while intoxicated.
c. Yes, because the cab driver’s character is a material issue in the litigation.
d. Yes, because the cab driver is not a defendant in this case.

A

c. Yes, because the cab driver’s character is a material issue in the litigation.

91
Q

In order to avoid paying high disposal fees, a factory illegally dumped its hazardous waste into a nearby reservoir. Upon discovering this fact, a woman who had taken a swim in the reservoir a month earlier met with a doctor out of precaution. During the examination, the woman told the doctor, “Last month my arms and legs were covered in rashes.” The doctor advised the woman to come see him again if the rashes recurred. In the factory’s subsequent criminal trial for illegal dumping, the doctor, as a witness for the prosecution, testified about the woman’s statement.

Upon the factory’s hearsay objection, will the court allow the doctor’s testimony?

A No, because the testimony contained privileged information.
B No, because the testimony was being offered as substantive evidence.
C Yes, because the woman was describing a then-existing condition.
D Yes, because the woman was seeking medical treatment.

A

The correct answer is: Yes, because the woman was seeking medical treatment.

Discussion of correct answer: Under the exception for statements made for medical diagnosis or treatment, the rule against hearsay does not exclude a statement that is made for medical diagnosis or treatment and that describes medical history, past or present symptoms, their inception, or their general cause. Here, the woman described her past symptoms to the doctor to obtain medical treatment. Although the woman’s out-of-court statement is being offered for its truth, it is admissible under this exception to the hearsay rule. Therefore, the court will allow the doctor’s testimony.

92
Q

After a large chemical spill at an industrial worksite, an inspector for the Environmental Protection Agency investigated the incident. In his initial visit, the investigator took pictures of the scene and made some notes after conducting interviews. After a three-week vacation, the investigator compiled his findings and submitted them in a report. Two weeks after submitting his report, he died of a heart attack. His conclusion was that the worksite owner and operator was negligent is allowing the spill to happen and suggested that fines and penalties be assessed.

In a subsequent civil case brought by a worker at the site against the owner and operator, the worker seeks to use the investigator’s report. The owner objects.

Is the investigator’s report admissible?

A No, because the report was not created contemporaneously with the chemical spill.
B No, because the report was created in anticipation of litigation, and the inspector is now unavailable.
C Yes, because the report was created by the inspector, who had a duty imposed by law.
D Yes, because the inspector is now unavailable.

A

The correct answer is:Yes, because the report was created by the inspector, who had a duty imposed by law.

Discussion of correct answer:Rule 803(8) creates a hearsay exception for most public records and reports. Properly certified official records from public offices are generally admissible if they are routine, factual, based on personal knowledge of public officials, and appear reasonably reliable. Investigative reports, reports with recommendations, and one-time reports prepared for a narrow purpose are generally admissible except against a defendant in a criminal case. No witness is required if the document is certified; it will be considered self-authenticated under Rule 902. Here, the report by the EPA inspector qualifies as a public record because it was created by a duty imposed by law, and therefore, it will be admissible.

93
Q

A student is on trial for burglary and conspiracy to commit burglary after he and his friend were caught carrying items away from their resident advisor’s room in their college dormitory. A witness can testify that the friend told her after the two were caught that stealing from the resident advisor was all the student’s idea, and that he had only gone along with it because he wanted to be friends with the student. The student argued that the statement is inadmissible hearsay. The friend was under house arrest awaiting his own trial.

Will the court admit the statement into evidence?

A Yes, because it is a statement against interest.
B Yes, because it an opposing party statement by a co-conspirator.
C No, because it is inadmissible hearsay.
D No, because doing so would violate the friend’s Fifth Amendment rights.

A

The correct answer is: No, because it is inadmissible hearsay.

Discussion of correct answer: This statement is not an opposing party statement by a co-conspirator because it was not made in furtherance of the conspiracy. It also cannot come in as a statement against interest, because that hearsay exception requires the unavailability of the declarant, and the declarant is available here as he is under house arrest. Because the statement is hearsay which does not fit within any of the exceptions, it will be inadmissible. As a test tip, timing is usually very important in determining whether a hearsay exception should apply in order to determine if the person who made the statement had a reason to lie. In this case, it was not an opposing party statement by a co-conspirator, because it was not made in furtherance of the conspiracy but rather, was made when he was already caught and had every reason to lie.

94
Q

A local surgeon was charged with manslaughter for causing the death of a patient. The prosecution alleged that although the surgeon was “on call” for the evening, he attended his brother’s wedding reception where he consumed numerous glasses of French champagne. The prosecution also alleged that due to his intoxication and the resulting impairment of his surgical skills, the patient died during an emergency appendectomy.
At the grand jury proceeding a doctor testified on the surgeon’s behalf. The doctor had been present at the same reception and had assisted during surgery. The defense called the doctor at trial. During direct examination, the defense attorney asked, “Isn’t it true that you saw the surgeon drink only soda during the wedding reception?” The doctor answered in the negative and stated he saw the surgeon drink at least four glasses of champagne.

The defense attorney then assailed the testimony as a recent fabrication and asked the doctor if he had not told the grand jury that he had only seen the surgeon drink ginger ale during the reception. The prosecutor objects. Will the prosecutor’s objection be sustained?

a. Yes, because a party may not impeach his own witness.
b. Yes, because the surgeon’s attorney has not admitted evidence of that prior statement.
c. No, because a party can impeach his own witness by a prior inconsistent statement.
d. No, because the doctor’s prior statement is admissible under the former testimony exception to the hearsay rule.

A

c. No, because a party can impeach his own witness by a prior inconsistent statement.

95
Q

A man was charged with domestic abuse of his wife after police responded to a call from his house. When they arrived, the man’s wife told police that her husband had beaten her. While the woman did not testify at the man’s trial, the police officer did testify about what she had told him. The man’s attorney objected to the admission of the wife’s statement to the police.

How should the trial court rule?

A The statement is inadmissible, because in criminal cases, all evidence presented against the defendant must be subject to cross-examination in front of the trier of fact.
B The statement is inadmissible, because it was testimonial in nature.
C The statement is admissible, because it was made while under the stress of a startling event.
D The statement is admissible, because it was not testimonial in nature and qualifies as an excited utterance.

A

The correct answer is:The statement is inadmissible, because it was testimonial in nature.

Discussion of correct answer:The Confrontation Clause of the Sixth Amendment, as interpreted in Crawford v. Washington [541 U.S. 36 (2004)], does not apply to nontestimonial statements not intended to be preserved as evidence at trial. In Hammon v. Indiana [547 U.S. 813 (2006)], the Supreme Court ruled that the victim’s statements to the police were testimonial. At the time of her questioning, she faced “no emergency in progress” and “no immediate threat to her person.” Instead, the relative safety of the conversation between the declarant and the officer made it “formal enough” to qualify as a “testimonial” statement intended as evidence of the past crime.

96
Q

A widow sued a neighbor for the sudden death of her husband. When he died, the husband had been hospitalized for injuries he suffered when he allegedly fell over a hose spread across the sidewalk in front of the neighbor’s house. On cross-examination of the emergency room physician who treated the husband in the hospital, the neighbor’s lawyer asked, “Don’t your records show that the deceased told you he had been a bit tipsy before he fell and landed on his head on the neighbor’s sidewalk?” The widow’s lawyer objected.

How should the court rule?

A The husband’s statement should be admissible only for impeachment purposes.
B The husband’s statement is admissible hearsay because it is a statement of cause reasonably pertinent to diagnosis or treatment.
C The husband’s statement is inadmissible hearsay.
D The deceased’s statement is inadmissible because the declarant is dead.

A

The correct answer is: The husband’s statement is admissible hearsay because it is a statement of cause reasonably pertinent to diagnosis or treatment.

Discussion of correct answer: Though it is hearsay, this statement is admissible under Federal Rule of Evidence 803(4) as a statement for the purpose of medical diagnosis or treatment. This exception allows not only a description of past symptoms and medical history, but also comments regarding the cause of the medical condition “insofar as reasonably pertinent to treatment.” Alcohol use would be pertinent to the cause of his medical condition and would therefore be admissible under this exception.

97
Q

A real estate agent is on trial for insurance fraud. The prosecutor seeks to offer testimony of the real estate agent’s husband that before their wedding, the real estate agent told him she had concocted a plan to help them come up with a down payment for a new house by claiming that she had been injured in a recent car accident, even though she had actually walked away from the accident uninjured. The husband is planning to divorce the real estate agent because of her dishonesty, but he has not yet filed the divorce papers. The real estate agent objects to the husband’s testimony. Can the real estate agent prevent the husband from testifying about her insurance claim plans by asserting the marital communication privilege?

a. No, because the pending divorce due to the real estate agent’s dishonesty indicates the two did not share a confidential and trusting relationship.
b. No, because the real estate agent made the communication before she was married to the husband.
c. Yes, because the marital communication privilege extends to communications made before marriage, as long as the parties subsequently marry.
d. Yes, because the parties are still married.

A

b. No, because the real estate agent made the communication before she was married to the husband.

98
Q

The plaintiff sued the defendant for injuries suffered in an automobile crash. The plaintiff claimed serious injury to her back. At trial, the defendant asserted that the plaintiff had a preexisting back injury at the time of the accident. The defendant called the plaintiff’s doctor to the stand. The doctor proposed to testify that when he examined the plaintiff after the accident, she claimed that her back was injured when the defendant failed to stop at a stop sign and crashed into her car. The plaintiff also told the doctor that she was in the process of suing the defendant, and that the plaintiff’s old back injury had not completely healed prior to the accident. The plaintiff objected to the testimony of the doctor.

Is the doctor’s testimony admissible?

A No, as hearsay not within an exception.
B No, because the probative value is outweighed by unfair prejudice.
C Yes, as statements made for the purpose of medical diagnosis or treatment.
D Yes, as nonhearsay.

A

The correct answer is: Yes, as nonhearsay.

Discussion of correct answer: This is a tricky Evidence question. Students must be aware that the bar examiners often test the rules about the opposing party’s statements–statements of a party offered by the opponent party. Here, the question attempts to lead students toward the hearsay exception for statements made for medical diagnosis or treatment. But, it is unnecessary to consider a hearsay exception, because opposing party’s statements are nonhearsay and freely admissible. Students would do well to approach hearsay questions by first asking if the declarant’s statement is one of a party that is being offered by the opponent. If so it is admissible without reliance on a hearsay exception.

99
Q

A landscaping company sent a worker out to maintain the trees on a woman’s property. The worker was very friendly and enthusiastic about trees but seemed not to know what he was doing. Several weeks later, one of the woman’s beloved trees began to shed its leaves. She had a specialist come to evaluate the tree out, and the specialist determined that the landscaping company must have treated the tree with a toxic herbicide, rather than a pesticide, and as a result the tree was dying and needed to be cut down. The woman called the landscaping company to complain but the owner did not want to talk to her. The owner then fired the worker effective immediately. Several weeks later, the worker came to the woman’s house in tears and said that it was all his fault that the tree had died, and that he realized later he had used herbicide rather than pesticide on the tree. The woman sued the landscaping company for the property damage and now seeks to admit the worker’s statements to her. The worker has offered to testify at trial.

Will the statements be admissible?

A Yes, because they were a statement against interest.
B Yes, because they were a present sense impression.
C No, because the statements were not against the worker’s proprietary or pecuniary interest.
D No, because the statements were not made while the worker was employed by the company.

A

The correct answer is: No, because the statements were not made while the worker was employed by the company.

Discussion of correct answer: Statements made by agents or employees of a defendant during the scope of that relationship may be admissible as non-hearsay statements of the opposing party, but this is only true during the scope of the agency or employment relationship. Here, the employee had been fired prior to his making the statements. No other exceptions apply, so the statements are non-admissible hearsay. As a test tip, when deciding if a hearsay statement will still be admissible as a hearsay exception or a non-hearsay statement, you should make sure that all the elements of either of those are satisfied by the facts. Timing is key for the statement to be a vicarious employee statement that can be used against the employer, because it is only fair to use the employee’s statement against the employer when the employee is still working for the employer. Once the employee leaves or is fired, there is a much greater chance that the employee has a grudge and is lying.

100
Q

A man sued his company claiming that he was denied his yearly bonus. The CEO claimed that the man was not a bonused employee. To prove this fact, he wished to introduce a record of all bonused employees. The CEO called the secretary who was in charge of payroll to testify to this record. The man objected claiming the report was hearsay.

How should the court rule?

A The testimony is inadmissible under the Confrontation Clause.
B The testimony is inadmissible, because the business record exception applies only to an actual entry in a business record.
C The testimony is admissible as proof of the absence of an entry in a business record.
D The testimony is admissible as habit evidence, provided there is corroborating evidence.

A

The correct answer is: The testimony is admissible as proof of the absence of an entry in a business record.

Discussion of correct answer: In order to prove who was owed a bonus, the secretary must testify to a document that lists all of the bonused employees. Because this list is an out-of-court statement being offered for its truth, an exception must exist for it to be admissible. Here, since the CEO is attempting to prove that the man is not on the list, and therefore not owed a bonus, the absence of an entry in a business record is what is being proved. The elements are similar to the business record exception; however, it is being used to show what is not there, rather than what is. Because the man’s absence from the bonus list is what is being sought to be proved, this exception provides for the list’s admissibility.

101
Q

A man lived next door to a woman. Late one night, the man overheard the woman scream “Ellen, put down the knife, please don’t do it!” The next day, the woman was found dead in her bedroom. Ellen was arrested and charged with murdering the woman. At trial, the man proposed to testify to the woman’s statement.

Upon objection by the defendant’s counsel, how should the court rule on the man’s testimony regarding the woman’s statement?

A Admissible as a dying declaration, if the jury determines that the woman believed that her death was imminent.
B Admissible as a dying declaration, if the judge, by a preponderance of the evidence, determines that the woman believed that her death was imminent.
C Inadmissible, because the probative value is substantially outweighed by the danger of unfair prejudice.
D Inadmissible, as hearsay not within any recognized exception.

A

The correct answer is: Admissible as a dying declaration, if the judge, by a preponderance of the evidence, determines that the woman believed that her death was imminent.

Discussion of correct answer: Under FRE 104(a), “Preliminary questions concerning the qualification of a person to be a witness, the existence of a privilege, or the admissibility of evidence shall be determined by the court.” The judge is not bound by the rules of evidence at this point, except those with respect to privileges. To the extent that these inquiries are factual, the judge acts as a trier of fact. The judge’s decision is final and is not subject to a contrary determination by the jury. On the other hand, the jury is to determine how much, if any, probative value or “weight” to accord to the admitted evidence, as well as to decide issues of credibility (i.e., whether to believe, wholly or in part, the witness’s testimony). The judge not only decides factual issues, he also determines the applicability of any technical evidentiary rules. He decides, for example, if a dying declarant had a sense of impending death; if an entry was made promptly in the regular course of business; if there was the necessary state of excitement to qualify a declaration as an excited utterance; if a witness is unavailable; or if an original document is unavailable so as to justify the admission of a copy under the best evidence rule. Finally, note that preliminary questions of fact are ordinarily resolved in both criminal and civil trials by using a preponderance of the evidence standard. The witness’s testimony as to the victim’s statement will be admissible as a dying declaration if the judge, by a preponderance of the evidence, determines the victim believed her death was imminent. Thus, this answer choice is correct.

102
Q

The defendant was on trial for murder. As part of his case-in-chief, the defendant called a friend to testify that he had known the defendant for 25 years and, in his opinion, the defendant was a peaceful man. On cross-examination of the friend, the prosecutor asked the friend the following question: “Are you aware that the defendant has been in three fistfights in the past 12 months?” The witness responded: “No, I did not know that.” Defense counsel objected.

Is the testimony admissible?

A No, as an improper means of rebutting character evidence.
B No, as improper cross-examination.
C Yes, for impeachment only.
D Yes, as substantive and impeachment evidence.

A

The correct answer is: Yes, for impeachment only.

Discussion of correct answer: Although it is the general rule that character evidence is inadmissible, there are instances when character evidence is admissible to prove a relevant character trait. In a trial for murder, the defendant’s character as a peaceful man is admissible in the form of opinion or reputation evidence. The defendant’s offer of the friend’s opinion testimony was, therefore, admissible character evidence. Once the defendant opened the door to character evidence, the prosecutor could rebut by offering evidence of the defendant’s character for violence, and could do so through opinion or reputation evidence. That evidence would be admissible substantively. Here, however, the prosecutor was not attempting to rebut the friend’s character evidence. Rather, the prosecutor was attempting to impeach the credibility of the friend by showing that the friend’s opinion was either inaccurate or unfounded. The effort here was purely to impeach the credibility of the friend, which is freely admissible.

103
Q

A doctor is on trial for battery of a former patient. He claims that he was acting in self-defense, as the client became enraged at the doctor’s advice to no longer pursue his dream of attending law school. During trial, the doctor testified that he was warned by two of the patient’s friends that the patient had a short temper and to “beware.” The prosecution objected.

How should the court rule?

A The statement is inadmissible, as hearsay not within an applicable exception.
B The statement is inadmissible, as a self-serving statement.
C The statement is admissible, as it is not hearsay.
D The statement is admissible, under the present sense impression exception to the hearsay rule.

A

The correct answer is: The statement is admissible, as it is not hearsay.

Discussion of correct answer: In many civil and criminal cases, the jury may be required to determine the state of mind of a defendant based on circumstantial evidence. Thus, under the Federal Rules of Evidence, any statements or warnings heard or otherwise understood by a defendant are not hearsay, because they show a defendant’s state of mind when he heard the statement. If the issue is whether the defendant (or some other person) actually heard the statement or warning, the statement will be admissible nonhearsay because the truth or falsity of the out-of-court statement is immaterial. Here, the doctor’s testimony, that others had warned him that his patient has a short temper and to beware, could technically be regarded as involving out-of-court statements, but they are not being offered to prove the truth of the matter asserted. Instead, the fact that the doctor had been told of the patient’s short temper and to beware is relevant as to whether the doctor had a good-faith belief that force was reasonably necessary in defense.

104
Q

A student is arrested and charged with the arson of a local yoga studio. At trial, the prosecution offers evidence that when the student was arrested shortly after the crime was committed, he had a large amount of heroin hidden in the side pouch of his motorcycle. The student’s attorney objects to the admission of this evidence.

How should the court rule?

A The evidence should be excluded, because this type of evidence is admissible only to rebut evidence of good character offered by the defendant.
B The evidence should be excluded, because the probative value is substantially outweighed by the danger of unfair prejudice.
C The evidence should be admitted, to prove the student’s bad character.
D The evidence should be admitted, to prove the student’s propensity to commit crimes.

A

The correct answer is: The evidence should be excluded, because the probative value is substantially outweighed by the danger of unfair prejudice.

Discussion of correct answer: Under Federal Rule of Evidence 404, past crimes or other wrongful acts of a defendant are admissible as circumstantial proof of an element of the charged crime, such as motive, intent, absence of mistake, identity, or common scheme or plan (the “MIMIC” rule). Pursuant to Federal Rule of Evidence 403, evidence may be excluded if its probative value is substantially outweighed by the danger of unfair prejudice. In this question, possession of heroin does not appear to have any connection with the arson. Thus, evidence of the student’s possession of heroin is inadmissible, because it would be more prejudicial than probative.

105
Q

A defendant was on trial for drug distribution. The arresting officer was called by the prosecution to describe in detail the events that led to the defendant’s arrest. The officer described the night in question, where he was, and what he observed. He was asked to provide a list of what he confiscated from the defendant after he was arrested. The officer testified that the defendant had a large amount of money and many small packets of drugs in several pockets throughout his clothing. When asked specifically about the number of drugs packets, the officer was unable to recall. The officer was then shown his police report. The officer read over his report for several minutes. The prosecutor then asked if the officer was able to remember how many bags of drugs were taken from the defendant. The officer responded that he could not. At this point, the prosecutor directed the office to page three of his report and asked him to read the fifth line, which said, “Confiscated from the defendant’s jacket were three purple packets with an off-white powdery substance, and confiscated from his right front pants pocket were 33 purple packets with an off-white powdery substance, that were placed on property receipt #43567.” The defense attorney objected to the officer’s testimony.

How should the court rule?

A Sustain the objection, because the officer lacks a present memory of the incident.
B Sustain the objection, and require the report to be admitted because the jury is just as capable as reading the report as the officer.
C Overrule the objection, because the officer’s memory was refreshed.
D Overrule the objection, because the officer’s testimony is admissible as a past recollection recorded.

A

The correct answer is: Overrule the objection, because the officer’s testimony is admissible as a past recollection recorded.

Discussion of correct answer: The attempt to refresh the officer’s memory was unsuccessful. Therefore, the prosecution needs to introduce the evidence by laying a foundation of the officer’s report, and then have that portion of the report read into the record. Although the out-of-court statement is being offered for its truth, the past recollection recorded hearsay exception will provide a means of admissibility.

106
Q

A bouncer is on trial for the rape of a waitress. On the night in question, the waitress stated during a 911 call that the bouncer had forced her to have sex at knife-point. However, when the bouncer was arrested at the scene of the crime, no knife was recovered. The bouncer offers the testimony of the waitress’s good friend that the waitress is sexually obsessive and has sex with a different man at least three times each week. The prosecution objects.

How is the court likely to rule?

A The evidence will be admitted, because it is relevant to the issue in the case.
B The evidence will be admitted, because an expert could give a foundation about obsessive behavior.
C The evidence will be excluded, because it violates the rape shield law.
D The evidence will be excluded, because the evidence is unfairly prejudicial.

A

The correct answer is: The evidence will be excluded, because it violates the rape shield law.

Discussion of correct answer: In a criminal case, barring certain limited exceptions, evidence of a rape victim’s prior sexual conduct or sexual predisposition is inadmissible. Pursuant to these exceptions, such evidence is admissible to prove that a person other than the accused is the source of semen, the victim’s injury, or other physical evidence, or if exclusion of the evidence would violate the constitutional rights of the defendant. Specific instances of sexual behavior between the accused and the victim may also be offered to prove consent. Because none of these exceptions apply to the proposed testimony, it is inadmissible.

107
Q

A waiter’s dog was stolen from his backyard one Saturday afternoon. A neighbor was charged with the theft. At the neighbor’s trial, the waiter testified that he happened to glance out his back door that afternoon just as the neighbor was unhooking the dog’s chain and dragging the dog from the yard. The neighbor’s attorney attempts to introduce evidence of the waiter’s own forged check conviction three years before, for which the waiter served the maximum sentence of six months. Is the evidence of the waiter’s conviction admissible?

a. No, because it was not punishable by imprisonment for over one year.
b. No, if offered on cross-examination for impeachment.
c. Yes, if offered on cross-examination, because the conviction involved dishonesty or a false statement.
d. Yes, to prove motive to commit the present crime.

A

c. Yes, if offered on cross-examination, because the conviction involved dishonesty or a false statement.

108
Q

A fan attended a minor league hockey game in his hometown. Unfortunately, he was only able to obtain tickets in the visitor’s section. While at the game, he became involved in an altercation with a fan of the visiting team. When the fan cheered for a home team goal, the visiting fan turned around and threatened to kill the home fan if he didn’t shut up. The home fan pulled a knife out of his pocket and stabbed the visiting fan in the arm. At his trial for aggravated assault, the home fan wants to introduce a statement from a witness who was standing next to the visiting fan at the game. The statement, which the witness had made earlier in the game when the home fan cheered for the home team, was, “You’d better watch out. At a hockey game last week, the visiting fan put two guys in the hospital when they wouldn’t shut up. One of them had 33 stitches after the visiting fan bashed his head against the steps.” Assume that the witness’s statement is being offered as proof of the effect it produced in the home fan’s mind.

In this regard, how will the statement most likely be found?

A Admissible, because the testimony is not hearsay as it is not being offered for its truth.
B Admissible, as a present sense impression.
C Inadmissible, as hearsay not within any recognized exception.
D Inadmissible, because the statement is self-serving.

A

The correct answer is: Admissible, because the testimony is not hearsay as it is not being offered for its truth.

Discussion of correct answer: This answer choice is correct, because the witness’s statement that the visiting fan had put two men in the hospital at a hockey game the week before is being offered to show its effect on the defendant’s state of mind, rather than being offered to prove the truth of the assertion. Thus, the witness’s statement bears upon the reasonableness of the home fan’s fear to justify his self-defense plea.

109
Q

Blue, an independent taxi driver, took his car to Aamco Transmissions to have the transmission repaired. Owens, Aamco’s employee, was working on the transmission when the hydraulic lift broke. This caused the car to plummet on top of Owens, injuring him. Blue, who happened to be standing nearby, saw the incident and rushed over to help Owens. As Blue tried to free Owens from the metal entanglement, the hydraulic pump (an apparatus connected to the lift) suddenly burst, propelling a sharp metallic fragment into Blue’s stomach. Blue received a deep laceration wound but was not otherwise injured. Blue then initiated suit against Aamco based on negligent maintenance of the hydraulic lift. At trial, Blue called the chief inspector for the state Department of Occupation Safety and Health Administration (OSHA), who testified about an official report of the accident as prepared by an OSHA investigator. Blue proposes to introduce a segment of the report that quotes Owens’ statement, made to the OSHA investigator in an interview at Aamco’s shop, as to the cause of the accident.

Should the trial judge admit the report over Aamco’s objection?

A Yes, as a past recollection recorded.
B Yes, as an opposing party’s statement incorporated in a public record.
C No, because such documents must be offered in their entirety.
D No, because it is hearsay not within any exception.

A

The correct answer is:

Yes, as an opposing party’s statement incorporated in a public record.

OR

Yes, as a party admission incorporated in a public record.

Discussion of correct answer: A vicarious statement is a statement by an employee or agent made within the scope of the employment relationship and which is now being offered against the employer. The facts state that Owens, Aamco’s employee, made a statement to the chief inspector for OSHA. That statement was contained within an OSHA accident report. Because the OSHA accident report qualifies as a public record, this is the correct answer.

110
Q

The defendant is on trial for bank robbery. The prosecution called the bank teller to testify. During his testimony, the prosecutor asked the teller the date of the robbery. The witness could not remember the date. The prosecutor then handed the witness a birthday card, asked the witness to look at the item, and then asked the witness if his recollection was now refreshed. The witness said yes, that the robbery occurred on his birthday. Defense counsel objected to the use of the birthday card. May the card be used to refresh the witness’s recollection of the date?

a. No, this is an improper way to prompt the witness’s testimony.
b. No, this is improper bolstering.
c. Yes, if the jury is allowed to view the card.
d. Yes, because the witness could not remember the date.

A

d. Yes, because the witness could not remember the date.

111
Q

The plaintiff in a personal injury action was riding her motorcycle on a country road when the defendant pulled his tractor out of a corn field onto the roadway. The plaintiff swerved off the road to avoid the tractor. The plaintiff suffered minor injuries and her motorcycle was severely damaged. The plaintiff sued the defendant for her medical expenses and repair costs. At trial, the defendant testified that the plaintiff was speeding at approximately 60 mph in a 35 mph zone. On cross-examination of the defendant, the plaintiff’s attorney asks, “Isn’t it true that you were convicted of perjury seven years ago?” The defendant denies that he was convicted of perjury. The plaintiff’s attorney then offers into evidence a certified copy of the defendant’s conviction to establish that he was found guilty of perjury seven years ago. Is this evidence admissible?

a. No, because character evidence cannot be admitted in civil actions.
b. No, because such extrinsic evidence cannot be used to prove specific instances of conduct.
c. Yes, because criminal convictions within the previous 10 years are admissible.
d. Yes, because this kind of extrinsic evidence may be offered to impeach the defendant’s veracity.

A

d. Yes, because this kind of extrinsic evidence may be offered to impeach the defendant’s veracity.

112
Q

A bookie was arrested and charged with the murder of a gambler. The bookie lived in New York City. The gambler, who resided in Philadelphia, was found shot to death in New York City. During the bookie’s murder trial, the prosecution called the gambler’s sister to testify. She proposed to testify that the day before the gambler was killed, he told her, “I intend to take the train to New York City tomorrow and pay my bookie the money I owe him.”

Is the sister’s testimony admissible?

A Yes, to show the gambler’s intent.
B Yes, as a verbal act.
C No, because the gambler is not subject to cross-examination.
D No, as hearsay not within any recognized exception.

A

The correct answer is:Yes, to show the gambler’s intent.

Discussion of correct answer:Federal Rule of Evidence 803(3) provides a hearsay exception, without regard to the availability of the declarant, for a statement of the declarant’s then-existing state of mind, emotion, sensation, or physical condition, including intent, plan, motive, design, or mental feeling. Such statements are admissible substantively to prove the declarant’s state of mind when that state of mind is at issue. The gambler’s statement will be admissible to show his state of mind the day before he was killed. Therefore, this answer choice is correct.

113
Q

A teenager was riding a motorcycle on a dirt road when a vehicle pulled out in front of him, causing the teenager to swerve and lose control of the motorcycle. The teenager was thrown off the motorcycle, sustaining severe injuries. The only eyewitness to the accident immediately called 911 and stopped to help the teenager. When the ambulance arrived about 15 minutes later, the eyewitness, who was still distraught over witnessing the accident, frantically explained to the paramedic how the accident occurred. The teenager sued the driver of the vehicle to recover damages for his injuries. Neither party could locate the eyewitness prior to trial. At trial, the teenager’s attorney calls the paramedic as a witness and asks the paramedic about the eyewitness’s statement to the paramedic, when the paramedic arrived at the scene, about how the accident occurred. The driver’s attorney objects on the basis of hearsay.

How should the judge rule on the objection?

A Overruled, because the eyewitness’s statement was a present sense impression.
B Overruled, because the eyewitness’s statement was an excited utterance.
C Sustained, because the eyewitness did not make the statement during or immediately after the accident.
D Sustained, because the eyewitness’s statement is hearsay.

A

The correct answer is: Overruled, because the eyewitness’s statement was an excited utterance.

Discussion of correct answer: Hearsay is inadmissible unless an exception applies. A hearsay exception exists for a statement relating to a startling event that is made while the declarant is under the stress or excitement caused by the event. The excited utterance exception may include statements that are made after the event so long as the declarant is still under the stress of excitement. Witnessing an accident where a person is thrown from a motorcycle and severely injured is a startling event. Although the statement by the eyewitness to the paramedic was made 15 minutes later, the facts state that the eyewitness was still distraught. Because the eyewitness’s statement was an excited utterance, the objection should be overruled.

114
Q

A CEO was addressing a large group of shareholders at an annual meeting in which his opening remarks discussed the health of the business, some recent acquisitions, and his personal health. “It was a long trip back,” he said, “but I am feeling great, ready to take on the world.” This news immediately hit the media and stock prices jumped 2%, as there was speculation that the CEO was suffering from severe depression.

The following morning, the CEO was found dead in his hotel room. After his death, the CEO’s wife attempted to collect on his life insurance policy, but her claim was denied due to speculation that the CEO committed suicide.

At trial, the wife sought to use the CEO’s statement to the shareholders that he was feeling great and ready to take on the world. The insurance company objected.

Should the trial judge admit the testimony?

A No, because there was speculation that he suffered from depression.
B No, because it was not made to a doctor.
C Yes, because it is relevant to the CEO’s medical condition.
D Yes, because it is a statement of the CEO’s then-existing physical condition.

A

The correct answer is: Yes, because it is a statement of the CEO’s then-existing physical condition.

Discussion of correct answer: FRE 803(3) allows for admission of an out-of-court statement describing the declarant’s then-existing mental, emotional, or physical condition. This includes a statement of the declarant’s then-existing state of mind (such as motive, intent, or plan) or emotional, sensory, or physical condition (such as mental feeling, pain, or bodily health), but not including a statement of memory or belief to prove the fact remembered or believed unless it relates to the validity or terms of the declarant’s will. Here, the proponent (the CEO’s wife) is attempting to show that her husband was not likely to have killed himself based on his comment the day before about how he was feeling. Because she wants to offer this statement for its truth and it discusses how the CEO was currently feeling, it falls within this exception.

115
Q

A fight broke out between two men at a sports bar, and one man later died from his injuries. At his criminal trial for homicide, the defendant called a witness to testify that he had seen the fight and that, from what he had observed, the deceased had been the aggressor. The witness also testified that he had worked with the defendant and the deceased for several years and, in his opinion, the defendant was a peaceful man, but that the deceased had a reputation for violence. The prosecution then called the deceased’s son to the stand, who proposed to testify that his father was a peaceful, non-violent person. The defense objected. Is the son’s testimony admissible?

A. No, because the defendant has not specifically questioned the deceased’s character.
B. No, because the deceased’s character may not be established by the testimony of his son.
C. Yes, to rebut the witness’s testimony that the deceased had a reputation for violence.
D. Yes, to rebut the defendant’s claim of self-defense.

A

C. Yes, to rebut the witness’s testimony that the deceased had a reputation for violence.

116
Q

A cook was employed at a restaurant, and his duties included keeping the kitchen clean. A patron got food poisoning after eating at the restaurant. The patron sued the restaurant. At trial, to prove that the kitchen was unsanitary, the patron’s attorney called the cook’s roommate as a witness to testify about statements that the cook had made to the roommate while the cook was employed at the restaurant regarding the cook’s unsanitary practices in the kitchen. The restaurant’s attorney objects to the roommate’s testimony.

How should the judge rule on the objection?

A Overruled, because the cook’s statements are against his interest.
B Overruled, because the cook was employed at the restaurant, and his duties included keeping the kitchen clean.
C Sustained, because the cook is not a party in the litigation.
D Sustained, because the cook’s statements are hearsay.

A

The correct answer is: Overruled, because the cook was employed at the restaurant, and his duties included keeping the kitchen clean.

Discussion of correct answer: Hearsay is an out-of-court statement offered to prove the truth of the matter asserted. However, a statement is not hearsay when the statement is offered against an opposing party, and the statement was made by the opposing party’s employee on a matter within the scope of employment. The cook was an employee of the restaurant, and during his employment, he made statements to the roommate regarding his unsanitary practices in the kitchen. The statements were within the scope of the cook’s employment because his duties included keeping the kitchen clean. Therefore, the statements are not hearsay.

117
Q

A plaintiff filed a suit against a defendant for injuries suffered when the defendant’s car struck the plaintiff as he was attempting to cross the street late one night. The plaintiff has testified that the defendant was driving without his lights. A passenger in the defendant’s car at the time of the accident, was called by the defense to testify that she had ridden with the defendant to work at night for five years and that the defendant always drove with his lights on at night. Should the passenger’s testimony be admitted?

a. No, because her testimony as an eyewitness is more relevant on the issue of whether, on this particular occasion, the defendant had his lights on.
b. No, because character must be established by reputation or opinion when offered to contradict direct testimony.
c. Yes, because prior conduct may be used to establish that the defendant is a careful driver.
d. Yes, because it is relevant to show that the defendant probably had his lights on at the time of the accident, as was his habit.

A

d. Yes, because it is relevant to show that the defendant probably had his lights on at the time of the accident, as was his habit.

118
Q

A girlfriend went to an amusement park for the day with her boyfriend. They were in line for a popular roller coaster when the girlfriend heard her boyfriend say nervously, “That operator just shut his eyes for a little too long.” A moment later the roller coaster, failing to brake, crashed into the rails at the entry gate and many riders were severely injured.

Many park patrons filed suit against the amusement park, alleging that it ignored regulations by overworking its employees and failing to give them proper breaks during their shifts. The suit alleged that the roller coaster operator was overtired and fell asleep, failing to apply the brakes at a critical time. The park claimed in defense that the outside maintenance company it had hired failed in its obligations to maintain the rides properly. The patrons sought to introduce the testimony of the girlfriend, who was no longer dating the boyfriend, about what she heard her boyfriend say.

Is her testimony admissible?

A No, because it is hearsay.
B No, because the boyfriend is able to testify himself.
C Yes, because the boyfriend made the statement at the moment he saw it.
D Yes, because the boyfriend was under the stress of the incident when he made the statement.

A

The correct answer is: Yes, because the boyfriend made the statement at the moment he saw it.

Discussion of correct answer: In general, a statement made out of court to prove the truth of the matter asserted is hearsay and is not admissible. There are, however, some exceptions to this rule. One such exception is for present sense impressions, where the declarant’s statement describing or explaining an event was made while he was observing it, or immediately after. Here, the boyfriend’s statement about the operator was made out of court and is offered to prove the truth of the matter asserted, as the patrons in the suit wish to prove that the operator was tired and was shutting his eyes. Therefore, unless an exception applies, his statement will be inadmissible hearsay. The present sense impression exception does apply, however, because the boyfriend’s statement describes what he was seeing and was made the moment he saw the operator shut his eyes.

119
Q

A student was taking a driving test with a driver education teacher from a private driving school. The student was driving and the teacher was in the passenger seat. Stopped first in line at a red light, the student began to roll forward. A pedestrian was crossing the street and was struck by the student. The pedestrian suffered multiple injuries, and sued both the student and the driving school. At trial, the pedestrian sought to establish that the light was still red when the student rolled forward, but the student claimed that the light was green. The pedestrian brought in as a witness a jogger who was on the sidewalk at the moment of the incident. The jogger planned to testify that the window in the driver education car was open and, although he didn’t notice the light himself, he heard the student say to the instructor, “I shouldn’t have moved…that light was red.”

Is this testimony likely to be admitted?

A Yes, as a statement of a party-opponent.
B Yes, because it is not being offered to prove the truth of the matter asserted.
C No, because the facts do not indicate that the teacher is unavailable to testify.
D No, because it is being offered to prove the truth of the matter asserted.

A

The correct answer is: Yes, as a statement of a party-opponent.

Discussion of correct answer: In general, an out-of-court statement offered to prove the truth of the matter asserted is inadmissible hearsay. Here, however, the student’s statement would be admitted as a statement of a party (the student), being offered by the opponent party (the pedestrian).

120
Q

A husband was arrested for physically abusing his wife, and was subsequently charged with aggravated battery. At the husband’s criminal trial on the charges, the prosecutor called a doctor to the stand to testify. The doctor had provided the wife medical treatment several days after she had allegedly been battered by her husband. The doctor was to testify that, when the wife had come to his office for medical treatment, the doctor had asked her how she had sustained her injuries, and the wife had replied, “My husband. He has a horrible temper and completely lost control.”

Is the doctor’s testimony admissible?

a. Yes, because the statement was made for purposes of medical diagnosis or treatment.
b. Yes, because the statement was a statement of personal or family history.
c. No, because the wife’s statement was protected by the physician-patient privilege.
d. No, because the statement was not reasonably pertinent to the diagnosis of the wife’s injuries.

A

d. No, because the statement was not reasonably pertinent to the diagnosis of the wife’s injuries.

121
Q

A driver was involved in a high-speed car accident. A truck owned by a trucking company ran the driver’s compact car off the road into a ditch. The driver was not hurt, but the side and front of his car were crushed. In a suit against the trucking company, the driver offers the testimony of a witness that after the truck driver ran into the driver’s car, the truck driver said, “I should have been watching the road.” The trucking company is the only defendant, and the truck driver is available to testify.

How should the trial judge rule on the testimony?

A Admissible, as a statement against interest.
B Admissible, as a nonhearsay vicarious statement.
C Inadmissible, because it is not a present sense impression.
D Inadmissible, as hearsay not within an exception.

A

The correct answer is: Admissible, as a nonhearsay vicarious statement.

Discussion of correct answer: Under Federal Rule of Evidence 801(d)(2), a statement is not hearsay if the statement is offered against a party and is a statement by his employee or agent (i.e., a vicarious statement) concerning a matter within the scope of his agency or employment and made during the existence of the relationship. The federal rules are liberal in allowing these vicarious statements, and there is a trend among state courts to allow them as well, even when the employee is not in the type of position generally recognized as an authorized spokesperson for the company.

122
Q

A local radio talk show host aired a show in which she stated that a local councilperson was a crooked politician. The councilperson was the subject of a recall effort that was ultimately successful, and he sued the radio station for defamation. The radio station defended on the grounds of truth. The councilperson took the stand and testified that he was not a dishonest politician. The station called a witness who testified that a year prior to the radio shows in question, the councilperson took a bribe in connection with a city redevelopment project at the municipal airport. The councilperson’s lawyer objects.
Should the court allow the testimony be admitted into evidence?

a. Yes, as relevant evidence of the councilperson’s character as a crooked politician.
b. Yes, as relevant evidence of character for truthfulness.
c. No, because it is inadmissible character evidence.
d. No, because the prosecutor may not introduce evidence of character unless the defendant puts his character in issue.

A

a. Yes, as relevant evidence of the councilperson’s character as a crooked politician.

123
Q

A man’s car broke down, and since he was only a mile from the repair shop, he decided to walk. As he approached the repair shop, he tripped on the cracked front cement step. The man fell and fractured his wrist. A woman who was jogging by rushed up to him and asked, “Are you hurt?” The man said, “Yes.” The jogger then exclaimed, “It’s horrible that a business would allow its front step to deteriorate to this state!” A few months later, the man filed an action against the repair shop to recover for his injuries.

Is the jogger’s statement admissible at trial?

A Yes, as a statement by an opposing party.
B Yes, as an excited utterance.
C No, as hearsay not within any recognized exception.
D No, as irrelevant nonhearsay

A

The correct answer is: Yes, as an excited utterance.

Discussion of correct answer: Under the Federal Rules of Evidence, an excited utterance is admissible as an exception to the general rule excluding hearsay evidence. An excited utterance is defined as a statement relating to a startling event or condition, made while the declarant was under the stress or excitement caused by the event or condition. A statement made by a witness to an accident will generally qualify as an excited utterance. In this case, the jogger rushed to help and expressed her concern, and was probably still under the stress of the event. Therefore, her statement is admissible pursuant to this exception to the hearsay rule.

124
Q

A heavyset man with long red hair robbed a liquor store. Thereafter, a man was arrested and charged with the armed robbery. At the man’s trial, the owner of the liquor store was called to testify. He admitted that he was unable to identify the man, who now had a shaven head, as the robber. The prosecuting attorney then handed the owner six photographs. He proposed to testify, over defense objections, that he had previously told the prosecuting attorney that picture #4, admittedly a picture of the man before he shaved his head, was a picture of the person who robbed his store.

How should the court rule on the owner’s proffered testimony?

A Admissible, as a prior identification by the witness.
B Admissible, as a past recollection recorded.
C Inadmissible, as hearsay not within any recognized exception.
D Inadmissible, because it is a violation of the man’s right of confrontation.

A

The correct answer is: Admissible, as a prior identification by the witness.

Discussion of correct answer: FRE 801(d)(1)(C) defines as nonhearsay a prior statement by a witness of identification of a person made after perceiving him, if the person making the identification is available to testify at the trial or hearing and subject to cross-examination concerning the statement. There is no requirement that the witness first be impeached, nor does a prior identification require that the identifying witness make a positive in-court identification. This choice is therefore correct, because the owner had previously identified the man in picture #4.

125
Q

A former federal court judge was being prosecuted for bribery after allegedly accepting $100,000 to rule favorably on behalf of a corporate defendant in an antitrust case. According to the prosecution, the judge was approached by the defendant’s agent, who offered him the bribe if he would dismiss the action. The judge agreed and accepted the $100,000, which he then deposited. Thereafter, the judge granted the defendant’s motion for summary judgment even though there was a genuine issue of material fact necessitating trial. At the judge’s criminal trial, the prosecution called a bailiff at the federal courthouse where the judge formerly presided to testify. The bailiff proposed to testify that “it was common knowledge at the courthouse that the judge was the best that money could buy.” Is the bailiff’s proffered testimony admissible?

a. Yes, because he had firsthand knowledge of the judge’s reputation.
b. Yes, because it is proper lay opinion.
c. No, because it is not the proper way to prove the judge’s character.
d. No, because the prosecution may not initially introduce evidence of defendant’s character.

A

d. No, because the prosecution may not initially introduce evidence of defendant’s character.

126
Q

A patient sued his doctor for malpractice. The patient called an expert witness to testify that the drugs prescribed by the doctor were so experimental that it constituted negligence under accepted practices in the medical community. On cross-examination, the expert stated that Pharmacology: A Guide for the Practitioner was a reliable authority in her area of specialty. At trial, it was also stipulated that the treatise in question was first published at the beginning of the last century and had been reprinted every year since with updates. The doctor’s attorney then proposed to enter the treatise as an exhibit, highlighting a passage which stated that the drugs prescribed by the doctor are widely used by other physicians in treating patients. The patient’s attorney objected.

Will the court admit the treatise?

A No, because the text of the treatise constitutes inadmissible hearsay.
B No, because the learned treatise exception requires the relevant sections be read to the jury, but not received as evidence itself.
C Yes, as hearsay falling under the learned treatise exception.
D Yes, because the jury determines the weight of the evidence.

A

The correct answer is: No, because the learned treatise exception requires the relevant sections be read to the jury, but not received as evidence itself.

Discussion of correct answer: FRE 803(18) provides that statements contained in treatises may be admitted into evidence during direct or cross-examination of an expert witness if: (1) the treatise is established as a reliable authority; and (2) the treatise is called to the attention of the expert witness during cross-examination or is relied upon by the expert in direct testimony. As an exception to the hearsay rule, statements in treatises admitted pursuant to FRE 803(18) may be used as substantive evidence. They may be read to the jury, but cannot be provided directly to the jury. Therefore, the treatise itself will not be admissible.

127
Q

A defendant was prosecuted for assault and battery after she admitted striking her sister with a broomstick that was part of her Halloween witch’s costume. The defendant claimed she acted in self-defense after she was attacked by her sister, who was calling her a “witch” and coming after her with a kitchen knife. As her first defense witness, the defendant calls a witness to testify that the defendant is a good solid neighbor.

Is the witness’s testimony admissible?

A Yes, because it is relevant to show the defendant’s character in this case.
B Yes, because it is relevant to support the defendant’s credibility.
C No, because it is merely the opinion of a character witness.
D No, because it is not directed toward a pertinent trait of the defendant’s character.

A

The correct answer is: No, because it is not directed toward a pertinent trait of the defendant’s character.

Discussion of correct answer: A defendant in a criminal case may offer character evidence on his own behalf, but the evidence here doesn’t qualify. Federal Rule of Evidence 404 provides that evidence of a person’s character or a trait of his character is not admissible for the purpose of proving that he acted in conformity therewith on a particular occasion, except when evidence of a pertinent trait of his character is offered by the accused, or by the prosecution to rebut the same. The witness’s testimony that the defendant is a “good solid neighbor” is not admissible because it is not a pertinent trait of her character. Here, the trait of being peaceful would be a pertinent trait, but being a good neighbor is not.

128
Q

A woman was indicted in federal court on charges that she illegally smuggled children into the country for adoption. There were eight other co-conspirators, including her husband. The woman objected to her husband serving as the principal witness against her. The district court ruled that the husband could testify to any act he had observed during their marriage and to any communication that was made “in the presence of a third person.” The court ruled that the husband could not testify to any confidential communications between him and his wife during the marriage. The woman appeals the ruling allowing her husband’s testimony. Should the appellate court affirm the ruling?

a. Yes, because only the husband as the witness spouse can assert the spousal privilege.
b. Yes, because the husband can waive any privileges relating to the marriage.
c. No, because the wife can assert the spousal privilege to block the husband’s testimony.
d. No, because the marital communication privilege applies.

A

a. Yes, because only the husband as the witness spouse can assert the spousal privilege.

129
Q

A plaintiff was seriously injured in a collision involving three trucks. A sideswipe occurred between the plaintiff’s pickup truck and a trailer truck as they passed in opposite directions. The sideswipe caused the plaintiff’s truck to careen down the road and into the path of a second trailer truck. The driver of the second trailer truck was immediately killed. Both of the trailer trucks were owned by the same transport company. At trial, the plaintiff called a witness to the collision to testify that he heard an unidentified female witness scream, “Oh my God, that trailer truck sideswiped that little pickup truck.”

How should the trial judge rule regarding the witness’s testimony?

A Admissible, as a declaration of then-existing state of mind.
B Admissible, as an excited utterance.
C Inadmissible, because it contains inadmissible opinion evidence.
D Inadmissible, as hearsay not within any recognized exception.

A

The correct answer is:Admissible, as an excited utterance.

Discussion of correct answer:Under FRE 803(2), the statement of the unidentified bystander, made in the presence of the witness, should qualify as an excited utterance or spontaneous exclamation within the meaning of that exception to the hearsay rule. The theory of this exception is simply that there has been some occurrence startling enough that temporarily stills the capacity of reflection and produces utterances free of conscious fabrication.

130
Q

After an airplane crash at a small airfield, an inspector for the Federal Aviation Administration investigated the crash. In her initial visit, the investigator took pictures of the scene and made some notes after conducting interviews. One particular interview was with a novice pilot who happened to be at the airfield when the crash occurred. He recalled to the investigator seeing something wrong with the left wing of the crashed plane before it took off, but that he didn’t have a chance to warn the pilot. He went on to say that “even a novice like him would have done a quick safety check and spotted the issue before taking off.” The investigator then compiled her findings and submitted them in a report. Her conclusion was that the pilot failed to conduct himself as a reasonably prudent pilot.

A ground worker at the airfield who had been burned trying to rescue the pilot brought a civil case against the pilot’s estate. The ground worker wanted to introduce the novice pilot’s statement from the investigator’s report. The estate’s attorney objected.

Is the novice pilot’s statement admissible?

A No, because it is hearsay not falling within any exception.
B No, because the estate does not have an opportunity to cross-examine the investigator or the novice pilot.
C Yes, as a business record, because the FAA routinely conducts such investigations.
D Yes, because it is contained inside of a public record compiled by a government agent under duty of law.

A

The correct answer is: No, because it is hearsay not falling within any exception.

Discussion of correct answer: Rule 803(8) creates a hearsay exception for most public records and reports. Properly certified official records from public offices are generally admissible if they are routine, factual, based on personal knowledge of public officials, and appear reasonably reliable. Investigative reports, reports with recommendations, and one-time reports prepared for a narrow purpose are generally admissible except against a defendant in a criminal case. No witness is required if the document is certified; it will be considered self-authenticated under Rule 902. However, statements by private citizens may not be included in official records unless they separately fall within a hearsay exception [D.W.S. v. L.D.S., 654 N.E.2d 1170 (Ind. Ct. App. 1995) (report from welfare department based on interviews of people who were under no duty to report did not qualify; person making report must be public employee with personal knowledge)]. Here, because the statement of the novice pilot is not independently admissible, it will not be admitted simply because it was written in a public record.

131
Q

Four men entered into a lease as tenants of a five-bedroom house. They each had a separate bedroom and used the fifth bedroom as a laboratory to manufacture the illegal drug methamphetamine.

One of the men was the so-called moneyman; he provided the funds to make the rental payments and to purchase the laboratory equipment. One of the other men was the chemist; he had both an undergraduate and graduate degree in chemistry and knew the formula and procedure for producing methamphetamine. Another of the men had sold drugs in the past and knew potential buyers of the methamphetamine. Finally, the fourth man was the enforcer; his role in the scheme was to make sure that no unauthorized persons entered onto the premises of the house.

The four men had been involved in this drug venture for seven months when they were finally arrested and charged with conspiracy to manufacture and distribute methamphetamine, and distribution and manufacture of methamphetamine. During the trial, the prosecution wishes to introduce, as evidence against the moneyman, a statement made by the enforcer to the landlord at the time of the signing of the lease, in which the enforcer said, “No matter what you charge us for rent, you better O.K. it with the moneyman, because he’s the one who will really be paying it.” The moneyman’s attorney objects.

The judge’s ruling on admissibility will depend on what fact?

A Whether the enforcer actually paid all of the rent.
B Whether the enforcer had a legal obligation to pay the rent under the terms of the lease.
C Whether the landlord may be considered a party-opponent.
D Whether the statement was made during the course of and in furtherance of the conspiracy.

A

The correct answer is:Whether the statement was made during the course of and in furtherance of the conspiracy.

Discussion of correct answer:If the enforcer’s statement was made during the course of and in furtherance of the conspiracy, the statement would be admissible as an opposing party’s statement. Under FRE 801(d)(2)(e), “a statement is not hearsay if the statement is offered against a party and is….a statement by a co-conspirator of a party during the course and in furtherance of the conspiracy.” The enforcer’s statement concerning the fact that the moneyman would be paying the rent for the premises where the methamphetamine was being manufactured would certainly fall within the “course of and furtherance of” the conspiracy to manufacture and distribute methamphetamine. All other statements would be incorrect as to prove the admissibility of the enforcer’s statement against co-conspirator, the moneyman.

132
Q

Olivia sued Jim for defamation after Jim wrote and published an untrue story about Olivia claiming that she used illegal drugs during school hours at the school where she was a teacher. Jim offered to settle the case, but Olivia was so enraged that she refused, and the parties proceeded to trial. During trial, Olivia’s co-worker Kate testified that she had never witnessed Olivia using drugs and, in fact, Olivia sponsored the anti-drug club at the school. On cross-examination, counsel for Jim asked Kate if two of Olivia’s brothers were heavy drug users who were able to keep their drug use a secret, which had not been addressed during Kate’s testimony on direct examination. Jim had a source who had given him information that this assertion was indeed true, and he expected Kate to say that she did not know; however, Kate answered with an unequivocal “no.” Jim would like to impeach Kate with evidence that Olivia’s brothers were indeed drug users.

Pursuant to Florida law, which of the following is correct regarding Kate’s answer to Jim’s last question?

Jim may impeach Kate by introducing extrinsic evidence to contradict her on the point of whether Olivia’s brothers are drug users.
Jim may impeach Kate by contradicting her on the point of whether Olivia’s brothers are drug users, but he may not use extrinsic evidence to do so.
Jim is bound by Kate’s answer, and he is not permitted to impeach her on the matter.
Jim may impeach Kate, but only by using reputation evidence, not opinion evidence.

A

Jim is bound by Kate’s answer, and he is not permitted to impeach her on the matter.

133
Q

During Mae’s criminal trial, the prosecution seeks to call several witnesses regarding Mae’s alibi. The prosecution is hoping to garner inconsistent statements from each witness, which will destroy their credibility and cast doubt on Mae’s alibi defense. In order for this plan to work, the prosecution does not want each witness to hear the testimony of the other witnesses, so the prosecution seeks to exclude each witness from the courtroom while the others are testifying. The prosecution’s motion will be:
Denied, because a party cannot motion to exclude anyone from the courtroom.
Granted, as long as the witnesses do not fall within into any exceptions for people who may not be excluded from the courtroom.
Granted, because anyone can be excluded from the courtroom
Denied, because the defendant has the right to have her alibi witnesses present in the courtroom.

A

Granted, as long as the witnesses do not fall within into any exceptions for people who may not be excluded from the courtroom.

134
Q

During a trial for damages arising from personal injuries sustained in a slip and fall accident, counsel for plaintiff, Helen, asks defendant Rich’s witness, Suzy, about the accident. Suzy was an eyewitness but now is unable to remember sufficient details to answer questions. Helen’s counsel seeks to introduce a memorandum of a statement made by a different eyewitness, Alex, as a recorded recollection. Assume that Suzy has adopted Alex’s statement. Rich’s counsel objects.

Sustained, because Suzy did not make the statement.
Sustained, because Florida does not provide for the admissibility of recorded recollections.
Overruled, because Alex’s memorandum is a proper recorded recollection.
Overruled, because Suzy adopted the statement made by Alex.

A

Sustained, because Suzy did not make the statement.

135
Q

A non-expert may testify as to the genuineness of handwriting as long as the testimony:

Is corroborated by at least one other witness.

None of the above.

Is not based on opinion that was formed in preparation for litigation.

Is in a civil trial, rather than a criminal trial.

A

Is not based on opinion that was formed in preparation for litigation.

136
Q

Paul, a prosecutor, is trying a case in which there are three joint defendants. Each defendant is given six peremptory challenges to use during the voir dire process. Paul is entitled to use:
Ten peremptory challenges.
No peremptory challenges, as these are reserved for the defendants.
Six peremptory challenges.
Eighteen peremptory challenges.

A

Eighteen peremptory challenges.

137
Q

At trial, the attorney for the defendant wants to affirmatively use the deposition testimony of a witness. The deposition may be used affirmatively in all of the following situations except:

If the deponent, whether or not a party, is dead.

If the deponent is an expert or skilled witness, but only if she is unavailable.

If the deponent is outside the state of Florida and thus beyond the subpoena power of the court, unless the party offering the deposition procured the absence.

If the deponent, whether or not a party, is farther than 100 miles from the place of trial, unless the party offering the deposition procured the absence.

A

If the deponent is an expert or skilled witness, but only if she is unavailable.

138
Q

Dick arranged to take two clients, Smith and Jones, deep sea fishing off the Florida coast. Jane, Dick’s wife, overheard a telephone conversation that Dick had with Smith about the fishing trip, but she never saw Smith or Jones, before she left home for work that day. Jane also knew that Smith had planned to pick Dick up at home early that morning. Jane comes home from work that evening and finds Dick on the couch shot in the leg. Dick murmurs, “Honey, I’m shot and I’ve been lying here for hours. Call Dr. Feelgood.” Several seconds later he says, “Smith shot me.” He then dies of a heart attack. At the criminal trial of Smith for the murder of Dick, Dick’s statement that Smith shot him is:
Admissible as a dying declaration.
Admissible as an excited utterance.
Admissible, because Dick is not unavailable.
Inadmissible, because it is hearsay.

A

Inadmissible, because it is hearsay.

139
Q

Jones was injured while examining a used car at Dealer’s car lot. Jones was looking under the hood of the car while a mechanic was working. The mechanic poured gasoline in the carburetor, which exploded when the mechanic tried to start the car, burning Jones badly. The mechanic made a routine written report of the accident in order to comply with the state safety regulations. Jones sues Dealer. The mechanic dies before trial. If Jones seeks to admit the mechanic’s properly authenticated report and the defense objects, the court should:

Overrule the objection; the report is not hearsay.

Sustain the objection; the report is hearsay.

Sustain the objection; the report is unreliable.

Overrule the objection; the report is a business record.

A

Overrule the objection; the report is a business record.

140
Q

In a trial, Judge Adams took judicial notice of the fact that City streetlights are lit at 7:00pm during the month of June. Which of the following is a correct statement?

Judge Adams has the discretion to determine whether the judicially noticed fact is conclusive, but only if the trial involves a civil action.

Judge Adams must instruct the jury that the judicially noticed fact is conclusive, but only if the trial involved a civil action.

Judge Adams has the discretion to determine whether the judicially noticed fact is conclusive.

Judge Adams must instruct the jury that the judicially noticed fact is conclusive.

A

Judge Adams has the discretion to determine whether the judicially noticed fact is conclusive.

141
Q

Dobb is being prosecuted for sexually assaulting Victim; however, Dobb claims that Victim consented to all of the alleged sexual activity. The prosecution calls Dobb’s wife, Wilma, and, after establishing her identity as Dobb’s wife, asked her what Dobb said on the night of the sexual assault. Dobb’s counsel immediately objected to Wilma’s testimony. The objection should be:

Overruled, because spousal immunity does not apply to criminal cases.

Overruled, because the only marital privilege recognized by Florida is the privilege for confidential spousal communications.

Sustained, because the privilege for confidential marital communications applies.

Sustained, because a wife cannot be forced to testify against her husband during the course of a criminal trial.

A

Sustained, because the privilege for confidential marital communications applies.

142
Q

Vlad died three days after a crane, operated by Drake, struck him. Vlad’s estate is now suing Drake. Counsel for the estate offers evidence that the day after the accident, Drake entered a drug rehabilitation center for treatment of cocaine addiction. The trial court would probably rule that such evidence is:

Inadmissible, due to physician-patient privilege.

Inadmissible, as irrelevant evidence.

Admissible, if offered to rebut Drake’s claim that Vlad was contributorily negligent.

Inadmissible, as against a subsequent remedial measure.

A

Inadmissible, as against a subsequent remedial measure.

143
Q

After she was sexually assaulted in a laundromat late one night, Janine met with a registered sexual assault counselor and told her about the incident. The counselor put Janine in touch with the police. Based on a description given by Janine, the police eventually arrested a Florida resident, Cameron, and Cameron was subsequently charged with rape.

If Cameron seeks an in camera review of Janine’s communication with the counselor, what is the standard that he will be required to meet under Florida law?

Cameron must demonstrate by a preponderance of the evidence that the privileged matters contain material information relevant to his defense.

Cameron must demonstrate a reasonable probability that the privileged matters contain material information necessary to his defense.

Cameron must demonstrate a substantial probability that the privileged matters contain material information necessary to his defense.

Cameron must demonstrate by clear and convincing evidence that the privileged matters contain material information relevant to his defense.

A

Cameron must demonstrate a reasonable probability that the privileged matters contain material information necessary to his defense.

144
Q

Zack was charged with murder by the state of Florida. His defense was that he was not in Orlando, the city where the crime occurred, on the date of the murder. At trial, the State offered into evidence an authenticated letter of Bill, the victim dated three days before his murder stating that he and Zack were headed to Orlando for a few days. Zack’s counsel objects.

How should the court rule on the objection?

Sustained, because Florida law provides that Bill’s future actions may not be proved through evidence of his then-existing state of mind.

Overruled, because Bill’s letter is a proper recorded recollection.

Sustained, because Bill’s letter is not a spontaneous statement.

Overruled, because Bill’s future conduct may be proved through evidence of his then-existing state of mind.

A

Overruled, because Bill’s future conduct may be proved through evidence of his then-existing state of mind.

145
Q

Sandy was the manager of the local bookstore. While devoted to her business, Sandy had poor record-keeping skills and did not make her staff fill out regular time sheets. Cal, an employee of the bookstore, did not get along well with Sandy. As a full-time employee, Cal was paid for 40 hours per week. However, Sandy felt that Cal was generally not very productive, and she decided to begin paying him for only 20 hours of work per week. When Cal received his next monthly paycheck, he protested the reduced amount. Sandy scribbled a note on a scrap of paper, saying, “Cal’s hours for October: 20 per week” and placed it in a file marked “Employee Hours.” There were no other notes in the file. Cal subsequently filed suit against Sandy and the bookstore to recover his unpaid wages for the month of October. At the trial on the matter, Sandy sought to introduce her note as evidence that Cal had worked only 20 hours per week during the month of October, under the “business record” exception to the hearsay rule.

Will the court allow the admission of Sandy’s note into evidence?

Yes, because Sandy had personal knowledge of the hours worked by Cal.

No, because the note is not a proper employee time sheet.

Yes, because the note qualifies as a business record even though it was handwritten.

No, because the note does not qualify as a business record.

A

No, because the note does not qualify as a business record.

146
Q

Mary, the owner of a restaurant, meets with Artie, an attorney, to discuss some business issues. During their conversation, Mary mentions to Artie that she would like to sell the building in which the restaurant was formerly located, but she is having trouble finding a buyer. The old location has been up for sale for several years and Mary needs the money to invest in the new location. Mary asks Artie some questions about how people get caught for insurance fraud, and ways to avoid it. A week later, Mary’s building that has been for sale burns down under suspicious circumstances, and she attempts to collect the insurance proceeds on the policy attached to the building. If Mary is later on trial for arson and insurance fraud, her discussions with Artie will be:
inadmissible, because they are protected by the attorney-client privilege.
inadmissible, because Mary did not directly inform Artie that she wanted to use this information to commit a crime.
admissible, because Mary sought information from Artie to commit what she knew was a crime.
admissible, because Florida does not recognize the attorney-client privilege.

A

admissible, because Mary sought information from Artie to commit what she knew was a crime.

147
Q

Vehicles driven by Elmer and Olive collide at a busy four way intersection. Before the filing of any lawsuit, Elmer tells Olive that he ran the stop sign and offers to settle the claim for $1500. Olive refuses to accept. Olive then sues Elmer for her personal injuries and property damage and Elmer, who was not injured, counterclaims for property damage. Olive testifies that her neighbor told her that his friend, a fireman, witnessed the accident and that the fireman, while still under the stress of the excitement of having viewed the accident, had told him exactly what he saw. Her attorney then asks Olive what the neighbor said to her about the accident. Before Olive can testify further, Elmer interjects a hearsay objection. The court should:
overrule the objection because the spontaneous statement exception to the hearsay rule applies.
sustain the objection if the fireman is not available to testify.
sustain the objection because the neighbor’s statement is hearsay and no exception applies.
overrule the objection because excited utterance exception to the hearsay rule applies.

A

sustain the objection because the neighbor’s statement is hearsay and no exception applies.

148
Q

On a party’s request during a trial, the court takes judicial notice that the town of Aurora is located beside a lake. The jury:

may disregard the fact, if it is a criminal case.

must accept the fact, if it is a civil case.

must accept the fact, regardless of whether the case is civil or criminal.

must disregard the fact, if another party has introduced conflicting evidence.

A

may disregard the fact, if it is a criminal case.

149
Q

A defendant was on trial for murdering his girlfriend. She was found shot to death in her bedroom. While the first officer to arrive was in the bedroom he saw the girlfriend’s open diary on her desk. The diary, which was confiscated by the police, was properly authenticated as being in the girlfriend’s handwriting. During the defendant’s murder trial, the prosecution called the officer to testify. He proposed to testify that when he looked at the open diary, he saw the following handwritten passage: “My boyfriend threatened to kill me today.”
Upon objection, is the officer’s proposed testimony admissible?

(A) 		Yes, because the diary was in plain view.
(B) 		Yes, because he has firsthand knowledge of the diary.
(C) 		No, because the best evidence rule requires the diary to be produced into evidence.
(D) 		No, because the passage in the diary is hearsay not within any recognized exception.
A

(D) No, because the passage in the diary is hearsay not within any recognized exception.

150
Q

Marina, a famous pop singer, is involved in a car accident while driving under the influence of alcohol in which she hit and severely injured Hannah, a pedestrian, outside of a Miami night club. Hannah brings suit against Marina alleging damages for her injuries, but Marina maintains that she was not at fault for the accident. Due to the negative press
surrounding the accident, Marina authorizes her spokesperson, Alana, to give a statement on her behalf. Alana addresses the press with the following statement: “Marina is extremely sorry for the injuries she caused to a pedestrian when she ran a red light while speeding and driving under the influence of alcohol last Saturday night.” If Hannah seeks to introduce Alana’s statement to prove Marina was at fault for the accident, the statement will be:

(A) inadmissible, because the statement was made by Alana, not Marina.
(B) inadmissible, because it is hearsay not within any exception.
(C) admissible, because it’s a statement of an opposing party.
(D) admissible, because it does not constitute hearsay.

A

(C) admissible, because it’s a statement of an opposing party.

151
Q

Chelsey is on trial for the murder of her husband Jeff. As part of her defense, Chelsey claims that she has an alibi, as she was at the country club at the time the murder allegedly took place. While no one saw Chelsey at the country club on the afternoon in question, the country club has a system that keeps track of each member who uses her access card to enter the building. Chelsey wants to prove she was at the country club by showing that her access card was used on the date in question. This evidence will likely be:

(A) inadmissible as hearsay.
(B) inadmissible, unless the evidence can be corroborated by an eyewitness.
(C) admissible, because this is a criminal, not a civil, trial.
(D) admissible, because it is a record of regularly conducted activity.

A

D) admissible, because it is a record of regularly conducted activity.

152
Q

Terry was badly beaten when he started making fun of some young men playing baseball in a Florida park, not realizing that the men were willing to use their baseball bat on him. As he lay in the hospital with many broken bones and internal injuries, he reasonably thought he was dying. Terry described the perpetrators to his roommate in the hospital. After about two months, Terry made a significant recovery and was released from the hospital. Around the same time, police made an arrest of two suspects in Terry’s beating, and he made the correct lineup identification. However, before the trials could take place, Terry crossed the street carelessly and was killed by a speeding truck. The prosecution could:

(A) have Terry’s identification to his roommate admitted under the present sense impression hearsay exception.
(B) have Terry’s identification to his roommate admitted under the hearsay exception for statement under belief of impending death.
(C) not have Terry’s identification to his roommate admitted, because it is hearsay not within any exception.
(D) not have Terry’s identification to his roommate admitted, because he was seriously injured when he made the statement and may have been on mind-altering drugs.

A

(B) have Terry’s identification to his roommate admitted under the hearsay exception for statement under belief of impending death.

153
Q

While Mary was driving her sister Sally to school, Mary’s car collided with one driven by Larry. That evening, at her attorney’s suggestion, Mary decided to write a summary of the accident while it remained fresh in her memory. Mary signed the document and showed it to Sally, who agreed to sign it as well. One month later, Larry sued Mary’s insurance company in Florida state court for compensation for his injuries. Because Mary had moved to Spain for school and would not appear as a witness, Larry called Sally to the stand and sought to have Mary’s summary admitted as Sally’s recorded recollection.

The court should rule that the summary as a recorded recollection is:

inadmissible, because the statement is hearsay.
inadmissible, because Mary made the statement, not Sally.
admissible, because Sally adopted Mary

s statement.
admissible, because Mary is not available.

A

inadmissible, because Mary made the statement, not Sally.

154
Q

A woman suffered a broken arm in a car accident. She brings suit, but needs an expert to testify as to causation. She consults a doctor and says “my arm is in tremendous pain, it broke due to striking it on my car door in a car accident, when the other car ran a stop sign.” At trial, the defense objects to this conversation.

What statements are admissible?

(A) None, because the doctor was retained exclusively for expert testimony, not treatment.
(B) The statement ”my arm is in tremendous pain.”
(C) The statement ”my arm is in tremendous pain, it broke due to striking it on my car door in a car accident.”
(D) The entire conversation.

A

(C) The statement ”my arm is in tremendous pain, it broke due to striking it on my car door in a car accident.”

155
Q

Defendant is on trial for conspiracy to commit bank robbery. As part of her case-in-chief the prosecutor offers into evidence the grand jury testimony of a witness who gave incriminating testimony about Defendant before the grand jury, but who has since died prior to trial. The prosecutor offers the original stenographic record of the witness’ sworn testimony before the grand jury. Defense counsel objects.

Should the court admit the testimony?

(A) No, because the witness’ grand jury testimony is hearsay not within an exception.
(B) No, because it represents a prior consistent statement.
(C) Yes, as former testimony.
(D) Yes, as past recollection recorded.

A

(A) No, because the witness’ grand jury testimony is hearsay not within an exception.

156
Q

A patient sued his doctor for malpractice. The patient called an expert witness. She testified that the drugs prescribed by the doctor were so experimental that it constituted negligence under accepted practices in the medical community. On cross-examination, the expert stated that Pharmacology: A Guide for the Practitioner was a reliable authority in her area of specialty. The expert testified that she, however, did not rely on that treatise in forming her opinion. The doctor’s attorney then proposed to read a portion from the treatise, which stated that the drugs prescribed by the doctor are widely used by other physicians in treating patients. The patient’s attorney objected.
Will the court admit the evidence?

(A) 		No, because the text of the treatise constitutes inadmissible hearsay.
(B) 		No, because the expert did not rely on the treatise in forming her opinion.
(C) 		Yes, as substantive evidence. 
(D) 		Yes, to impeach the expert, but not as substantive evidence.
A

(D) Yes, to impeach the expert, but not as substantive evidence.

157
Q

While riding his bike to school one morning, Eric is struck by a vehicle driven by Sophia. As a result of the accident, Eric is required to undergo a series of complicated surgeries. Sophia, who is extremely wealthy, learns that Eric’s family is poor and cannot afford the surgeries, so she offers to pay for them. Eric’s parents accept the offer, and allow Sophia
to pay for several of Eric’s surgeries. They then bring suit against Sophia for negligence in causing the accident. If they seek to introduce evidence of Sophia’s offer to pay, and subsequent payment of, Eric’s medical bills, the evidence will be:

(A) admissible with regard to Sophia’s offer to pay.
(B) admissible with regard to Sophia’s payment of Eric’s medical bills.
(C) admissible with regard to both Sophia’s offer to pay and Sophia’s payment of Eric’s medical bills.
(D) inadmissible with regard to both Sophia’s offer to pay and Sophia’s payment of Eric’s medical bills.

A

????

158
Q

As a result of a collision, Dustin is injured by a car being driven by Kimberly. Dustin has brought suit against Kimberly, alleging that she is liable for causing the collision which resulted in his injuries. Overcome with guilt, Kimberly sends a note to Dustin which says, “I am so sorry about the accident, and the fact that you are still in the hospital. I feel so bad for causing this accident.” If Dustin tries to introduce this note into evidence, he will be able to introduce:

(A) the part of the statement in which Kimberly admits she caused the accident.
(B) the entire statement.
(C) none of the statement.
(D) the part of the statement in which Kimberly says she feels bad that he is still in the hospital.

A

(A) the part of the statement in which Kimberly admits she caused the accident.

159
Q

Mary sued John for injuries she sustained falling on John’s staircase as she left his apartment building. John’s attorney seeks to introduce evidence that the day after the accident, John’s landlord, Alice, repaired the staircase. The evidence is:

(A) admissible, because it is evidence of a routine practice of an individual.
(B) admissible, because it is used to shift the blame to another party.
(C) inadmissible, because it is hearsay.
(D) inadmissible, because it is a subsequent remedial measure.

A

(B) admissible, because it is used to shift the blame to another party.

160
Q

Bernie consults his lawyer telling him that he is running an illegal investment scheme to defraud investors. He asks the lawyer advice as to how to avoid being caught. In a later fraud prosecution of Bernie,

(A) Bernie’s lawyer may claim privilege on behalf of his client.
(B) Bernie may claim privilege regarding confidential communications he had with his lawyer.
(C) Bernie may not claim privilege because the disclosure regarding the fraud is not confidential.
(D) Bernie’s lawyer may not claim privilege on behalf of his client.

A

?????

161
Q

After Robert was assaulted at a fraternity party, he filed a civil action against the fraternity and its president. Robert subpoenaed Bill, a writer for the school newspaper who wrote extensively about the incident on his personal website. Robert sought the admission of Bill’s notes from his interviews with several students who attended the party.

Which of the following, if true, would be Bill’s strongest argument in favor of journalist privilege?

The information can be obtained from an alternative source.

There is no compelling interest for requiring disclosure.

Bill has already drafted a book proposal based on his website.

Bill interviewed the students for a story for his newspaper.

A

Bill interviewed the students for a story for his newspaper.

162
Q

Jason hires Shelley to walk his dogs three afternoons a week while he is at work. After several weeks, Jason realizes that Shelley has been stealing small trinkets from his apartment each time she comes to walk his dog. Most of the items are insignificant in value, but one is an antique paperweight worth over $3,000. At Shelley’s trial for wrongful taking of the paperweight, Jason seeks to introduce a picture of the paperweight in lieu of the actual paperweight. Pursuant to Florida law, Jason will:

(A) not be permitted to introduce the picture, since the item can be easily transported to the court.
(B) not be permitted to introduce the picture, but may introduce a written description of the item.
(C) be permitted to introduce a picture of the item, as long as the photograph has certain information written on the back.
(D) be permitted to introduce a picture of the item, as long as he can provide corroborating testimony that this is indeed the paperweight that has been stolen.

A

(C) be permitted to introduce a picture of the item, as long as the photograph has certain information written on the back.

163
Q

Selby and Stefan, a married couple, live in Florida, where they support themselves by robbing stores. After a dozen or so robberies, they are apprehended and tried. The prosecutor puts pressure on Selby, encouraging her to testify against Stefan in exchange
for immunity, but Selby refuses. She seeks the advice of her lawyer as to what the prosecution can compel her to do. Her lawyer tells her that:

(A) she can be compelled to testify at Stefan’s trial, but she cannot be compelled to disclose marital communications.
(B) she cannot be compelled to testify at Stefan’s trial or disclose marital communications.
(C) she can be compelled to testify at Stefan’s trial, and the joint participants’ exception will apply.
(D) she cannot be compelled to testify or to disclose marital communications.

A

(A) she can be compelled to testify at Stefan’s trial, but she cannot be compelled to disclose marital communications.

164
Q

At the defendant’s trial for murder, the defendant sought to introduce the testimony of John, the defendant’s cousin, who was to testify that he was with the defendant out of state on the night of the murder. The prosecution then moved to introduce evidence that three years ago, John had been convicted of filing a false tax return. The defendant objected to the evidence of John’s conviction. Will the court overrule the objection?

(A) Yes, because evidence that a witness has been convicted of any crime is admissible to attack his credibility.
(B) Yes, because the crime of which John was convicted involved dishonesty or a false statement.
(C) No, because the crime of which John was convicted was not punishable by death or imprisonment in excess of one year.
(D) No, because the conviction occurred over one year ago and is of little probative value.

A

(B) Yes, because the crime of which John was convicted involved dishonesty or a false statement.